You are on page 1of 68

INTRODUCTION:

6 Themes of Professional Responsibility Conflicts of Interest a. Lawyers should have undivided loyalty to their clients. b. 1.7, 1.8, 1.9 (former clients) c. lawyer vs. client 2. Truthfulness a. Honesty: Rule 8.4 prohibits dishonesty, fraud, deceit and misrepresentation (Misconduct 8.1Applicant) b. 3.3 Candor toward tribunal: Youre an officer of the court! 3. Lawyers duty to client vs. duty to system (spectrum of client-centered vs. public-centeredmost lawyers are somewhere in the middle): Attention to the public interest (its not okay to just forward the interests of your client, you musts consider the public interest). a. Rule 1.4: Scope of representation 4. Lawyers personal/professional interests vs. fiduciary obligations: Attorneys have a unique position of TRUST 5. Self-interests (we are self-regulated- see comment number 12 in the preamble) a. We are self regulated, but how do we embrace our professional norms? b. Compare comment 12 in the preamble with Rule 1.5 (discretionary power over fees) which is ambiguous about attorneys fees, notifying clients about billing. Ex: Fen-Phen case 6. Institutional Pressures on professional judgments (rookie attorneys poor judgment will not be excused because they are inexperienced. Supervising attorneys can be held responsible as well for poor judgment on behalf of new attorneys) a. 8.3: Reporting misconduct 1. Often, but not always, the right thing to do in a particular situation will also comply with the ethical rules. Three questions to assess your conduct (when the right thing conflicts with the ethical thing to do): 1. Does the conduct in question violate the ethical rules? 2. Does it violate some other law (criminal, regulative?) (Along with the ethical rules, you have to worry about all other laws like the Constitution, statues, etc.). 3. What is the right thing to do? This isnt always going to be determinative of your conduct though.

CHAPTER 1: The Regulation of Lawyers


INSTITUTIONS THAT REGULATE LAWYERS: 1

1. The highest state courts: a. State and local bars: b.We have an Integrated bar: Admission to the bar in Kentucky: KY SCR 2.000 et seq: Bar application i. To be admitted, you must pass the MPRE (2.015), must have good moral character, must have approved bar application, etc. c.Inherent powers doctrine: Courts have the exclusive authority to regulate the bar. Chambers v. Stengell talks about balancing the police power and the exclusive authority of the courts to regulate the bar. d. How state courts regulate lawyers: i. adopts ethics codes and court procedural rules to govern lawyers ii. sets and implements standards for licensing of lawyers, including educational and moral character requirement iii. supervises agencies that investigate and prosecute complaints of unethical conduct by lawyers iv. supervises administrative judicial bodies that impose sanctions on lawyers who violate ethics codes 2. State and local bar associations: 3. Lawyer disciplinary agencies: 4. ABA: a private nonprofit membership organization founded in 1878. State bar associations are independent, not subordinate to the ABA. a. Model rules: 5. American Law Institute: 6. Federal and state trial courts: 7. Legislatures: 8. Administrative Agencies: 9. Prosecutors: 10. Malpractice insurers: 11. Law firms and other employers: 12. Clients: Elements of a lawyer-client relationship: 1. Competence (1.1) 2. Confidentiality (1.6) 3. Agency (lawyer as agent of the client) 4. Fiduciary: unique position of trust and confidence 5. Undivided loyalty and diligence 6. Duty to inform (Rule 1.4) communication Fen-Phen Attorneys illustrate a systemic and personal dilemma Preamble: 1, 9, 12, 1.4 (failure to communicate), 1.5, 3.3 Candor toward tribunal, 8.3 reporting, 8.4 misconduct CASES-BAR APPLICATIONS: 2

In re Prager: Intl drug smuggling, fled, extradited, probated, but graduated summa cum laude and clerked for state supreme court judge: Denied. Good acts insufficient to out balance 16 years of pot smoking and smuggling Vaughn v. Bd of B. Examiners: Sexual relations with students (applicant used to be a public schoolteacher): Denied, court said his ethical value system was deplorable Fla. Bd. of B. Examiners re S.M.D.: Declaration of bankruptcy after charging wedding, credit cards, etc. Admitted: record did not sufficiently demonstrate financial irresponsibility Application of Gahan: Declaration of bankruptcy after law firm he worked for could not pay him-discharged his 14,000 student loan: Court did not find moral turpitude or fraud, but denied admission because his hardship was no compelling and failure to repay loans showed lack of good moral character. In re Tobiga: Shoplifting of meat- many positive character witnesses: applicant found to have proven moral character. Friedman v. conn. B. Examiners: Student accused of cheating on law school exam. Denied. The student who claimed the cheating was more credible than the other student, and there was other evidence to suggest applicant lacked good character Radtke v. Bd. of B. Examiners: Plagiarism from a university teaching position was concealed, application denied. Allowed to re-apply after one year. The underlying misconduct was unconnected to any legal work. In re Converse: Pattern of hostile and disruptive conduct: nude caricatures, display of nude females backside in his study carrel, launched personal attacks. Denied. PROBLEM 1-1, POT: PAGE 57 During law school, youve smoked pot a few times. You need to disclose this. Taken from Iowa (most people are admitted). Rule 8.1: applicant shall not knowingly make a false statement of material fact. Cases illustrate how subjective the judgment is. It also illustrates how intrusive the bar admission is. In re Converse
Bar admission denied for law student engaging in provocative acts (pattern of hostile conduct)

In re Tobiga
Shoplifting for stealing meat and lied about unpaid loans; admitted

SMD
financial responsibility case, admitted

Mental health of applicants: The mere fact of treatment for mental health problems or addictions is not, in itself, a basis on which an applicant is ordinarily denied admission in most jurisdictions. Many states, until the 80s and 90s, asked very intrusive questions. Now most states have narrowed their questions to ask about mental illnesses that require hospitalization or that involved psychotic disorders. Rose Gower She spent two weeks in the hospital for depression during high school. She disclosed on her bar application, and then the bar committee delayed her admittance for more than a year with 3

continuous questions requiring more information about her medical history. (Discuss balance between safeguarding against inappropriate character/fitness and candor) In re Mustafa: Highest court in DC: Mustafa was a co-chief justice of the law schools moot court program and he took money out of the organizations bank account. It was during his third-year in law school. The other co-chief justice turned him in and he also turned himself in that same day. He claimed he took money out to pay his sisters bail and he did pay it back. Admission to bar denied because he failed to establish fitness. Mustafa later was admitted to practice in California, but was fired and sued for wrongful termination (clients claim he did not return phone calls, appear at hearings, used business funds for personal use, etc.) How was he able to practice in California after being denied admission in DC? If something happens in law school, as opposed to earlier in ones life, its regarded as more seriously as indicative of bad moral character clear and convincing evidence that you possess good moral character How can we prevent future misconduct? Improvements that could be made: Clients dont know anything about the disciplinary system the sanctions imposed are often light and innocent- private sanctions, which are the lightest form of discipline, are imposed almost twice as often as other sanctions. They fail to address primary goal of lawyer discipline, protection of the public. Problem 1-2: The doctored resume: Third-year law student from another country lied on her resume, changed transcripts, etc. Youre on the Honor Council: Should you expel her, determine some other punishment, or feel bad for her and let her stay. Options: Expel, violation of the honor code, warning (consensus: warning not stringent enough). She probably needs to correct the resume and disclose it to the bar, and possibly a note in her law school file. She cant be penalized under the bar yet since shes only a student- but shell likely be denied admission to the state bar. Naivity cannot be an excuse for lying.

CHAPTER 2: LAWYER LIABILITY:


Notion of self-reporting: Reporting is the key notion of self-regulation, but it can lead to underenforcement. You can be disciplined for conduct OUTSIDE of practicing law! a.The history and process or lawyer discipline i. Process: 1. Complaint filed by a client or lawyer 2. bar counsel investigates complaint

1. Professional Discipline:

a. If complaint does not warrant charge, filed closed, OR b. Charges filed if warranted by investigation i. hearing committee conducts hearing, makes factual findings, recommends sanctions ii. hearing committee decision reviewed by judicial agency and/or by highest state court. Reviewing body makes final decision on sanction. b. Grounds for discipline i. Tens of thousands complaints received annually are not addressed because there are so many. ii. In re Peters: Dean of William Mitchell College of Law was reprimanded for engaging in unwelcome physical contact and verbal communication of a sexual nature against four women employees. He was publicly reprimanded: case was made public and probably a notice was published in the bar journal announcing the decision. He wasnt publicly reprimanded for five years. Was this enough? No... 1. A lawyer can be disciplined for actions which are illegal but do not result in criminal conviction 2. Peters was eventually forced to resign from the law school but serves as president of an organization and was honored for his work with a charities organization a few years ago. 3. Deans and professors can be punished the same way lawyers are- they are still attorneys. iii. Common categories of questionable conduct by law professors: 1. sexual harassment 2. plagiarism 3. neglect of teaching responsibilities 4. manipulation of grades 5. aggressive or discriminatory behavior 6. dishonest behavior iv. Lawyers can be disciplined for violation of the ethics code whether or not the violation occurs in the course of law practice (any conduct that is dishonest or prejudicial to the administration of justice or that reflects lack of fitness to practice). Other conduct examples: 1. domestic violence 2. failure to pay child support 3. drunk driving 4. even for putting slugs in parking meters! 5. any crime that reflects dishonesty, untrustworthiness, or lack of fitness to practice v. If a lawyer is disbarred or suspended in one state, and they are licensed in more than one state, they have to report it to all the states they are licensed to practice in. vi. A lawyer can be disciplined for violating a rule in a state in which she is not licensed to practice. vii. A lawyer can be disciplined for something she does outside the state in which she is licensed to practice viii. A lawyer can be disciplined based on the actions of an employee. 5

c.Reporting misconduct by other lawyers i. The duty to report misconduct 1.Rule 8.3: a lawyer who knows that another lawyer has committed a violation of the rules of professional conduct that raises a substantial question as to that lawyers honesty, trustworthiness or fitness as a lawyer in other respects, shall inform the appropriate professional authority... a. A lawyer who knows of a violation by any other lawyer (an adversary, a partner, a boss, public official, another lawyer in their firm) must report it to the bar disciplinary agency i. Exceptions: only those raising a substantial question of the lawyers honesty, trustworthiness or fitness must be reported. ii. know: see rule 1.0: actual knowledge b. This includes reporting misconduct by judges c. does NOT require disclosure of information otherwise protected by Rule 1.6 (confidentiality rules) or info gained while participating in an approved lawyers assistance program d. The duty to report is triggered by knowledge of another lawyers misconduct. e. Standard for assessing knowledge is objective. The knowledge must be more than a mere suspicion that misconduct has occurred. Question is whether a reasonable lawyer in the circumstances would have a firm opinion that the conduct in question more likely than not occurred f. Do not have to report confidential client information under Rule 1.6 i. You could disclose if it wouldnt hurt the client and they give informed consent g. If you learn of misconduct during proceeding, you can wait until proceeding has concluded to protect a clients interests. h. This includes the duty to blow the whistle on your boss if she does something unethical. i. A lawyer may be subject to discipline if she fails to report serious misconduct by another lawyer. Although there are few public reports of this happening, the possibility of discipline for not reporting has motivated many lawyers to come forward with reports of unethical behavior. j. Daryl van Duch, Best Snitches: Land of Lincoln Leads the Nation in Attorneys Turning in Their Peers: (breaks down Conspiracy of silence) Illinois has staggering numbers of reporting (in 1995, nearly 600 complaints filed). Why? In 1988 the Ill highest court suspended an attys license for failing to report another lawyers misconduct. Facts: James Himmel helped client recover monies pocketed by first attorney, and never reported the first attys misconduct (the client asked him not to report it). He had a duty to report the attorneys wrongdoing and was therefore disciplined. k. The Himmel Rule also had negative repercussions- state had to make it unethical to threaten to snitch during pending civil litigation about opposing counsels alleged misconduct b/c too many lawyers were using skeletons to gain negotiation leverage. 6

l. Normally, in these situations you would not have to tell- here, the situation had been so public that the atty-client privilege was waived. m. Some states like NY have adopted a law firm rule to hold entire firms accountable for misconduct. ii. Lawyers responsibility for ethical misconduct by colleagues and superiors: (the ethics codes impose a limited amount of collective responsibility on other lawyers in the firm organization for the conduct of other lawyer and of non-lawyer employees) 1.Rule 5.1: explains responsibility of a partner or supervising atty for ensuring compliance with the ethical rules by subordinate lawyers, and explains when a senior lawyer may be subject to discipline for the conduct of a subordinate lawyer a.Rule 5.1(a): A partner, and a lawyer to individually or together with other lawyers possesses comparable managerial authority in a law firm shall make reasonable efforts to ensure that the firm has in effect measures giving reasonable assurance that all lawyers in the firm conform to the Rules of Prof Conduct i. lawyer managers must set up systems to prevent ethical problems (includes procedures to check for conflicts of interest and to manage client funds, providing continuing education in legal ethics). law firm includes legal services orgs or legal depts of corps, gvt agencies, etc. b. Rule 5.1(b): A lawyer having direct supervisory authority over another lawyer shall make reasonable efforts to ensure that the other lawyer conforms to the Rules of Professional conduct i. a supervising atty is not responsible for ethical violation of subordinate atty if they do not know about it. but the violation could be a breach of their supervisors duty under 5.1(b) if they dont make reasonable efforts to prevent the violation ii.SEE COMMENT 5 for Rule 5.1, Page 337 of Rules Supplement c. Rule 5.1(c): A lawyer shall be responsible for another lawyers violation if: i. the lawyer orders or, with knowledge of the specific conduct, ratifies the conduct involved; or ii. the lawyer is a partner or has comparable managerial authority in the law firm in which the other lawyer practices, or has direct supervisory authority over the other lawyer, and knows of the conduct at a time when its consequences can be avoided or mitigated but fails to take remedial action 2.Rule 5.2: explains when a subordinate lawyer is responsible for her own conduct, and when she can follow orders without fear or discipline a.Rule 5.2(a): A lawyer is bound by the Rules of Prof. Conduct notwithstanding that the lawyer acted at the direction of another person b. You cannot say, I was ordered to do it, thats why I did it!.

c. Rule 5.2(b): A subordinate lawyer does not violate the Rules of Prof Conduct if that lawyer acts in accordance with a supervisory lawyers reasonable resolution of an arguable question of professional duty. i. To know if the supervisors instruction is a reasonable resolution of an arguable question of professional duty, they should 1) do some research and 2) seek advice from someone more experienced. ii. SEE COMMENT 2 ON PAGE 341 OF RULES SUPPLEMENT 1. Someone has to make a judgment so when the supervisor does it, and turns out to be wrong, the subordinate is safe from discipline d. In most states, only individuals can be disciplined for violations. But in NY and NJ, there can be discipline on law firms for certain misconduct such as failure to supervise attorneys e.Problem 2-1: The Little Hearing: New associate thrown into a hearing for immigration law with no experience and little time to prepare. Page 109. Rule 5.2b applies b/c her supervisor might be the one that gets in trouble. 1.1 is also relevant: Competence. See Rule 1.1, Comment 1 and 5 (page 27 of Rules Supplement). Rule 5.1b also applicable. i. Comment 1: factors for knowledge and skill: 1. complexity/specialized nature of the matter 2. lawyers general experience 3. training and experience in the field in question 4. preparation and study the lawyer is able to give the matter and whether it is feasible to refer to matter to, associate or consult with, a lawyer of established competence in the field in question ii. Comment 5: Thoroughness and preparation factors: 1. inquiry into and analysis of the factual and legal elements or the problem 2. use of methods and procedures meeting the standards of competent practitioners 3. adequate preparation 4. what is at stake iii. she should either quit or report. Powell says there is a duty to report iv. 8.3 Integrity of the Profession: v. Rule 1.3 Diligence: A lawyer shall act with reasonable diligence and promptness in representing a client. 3.Rule 5.3: (language almost identical to 5.1) explains responsibilities of lawyers who supervise non-lawyer employees for ensuring they comply with rules of professional conduct. 4.Wieder case (NY 1992): law firms cannot dismiss attorneys for reporting misconduct. The requirement that lawyers with evidence casting substantial doubt on another lawyers honesty, trustworthiness or fitness inform disciplinary authorities is an implicit part of his employment contract with the law firm. Statute: When a firm discharges a lawyer for making such a report, that firm can be sued for breach of contract. Until this case, the law provided no such protection and associates could be fired for any reason other than one expressly prohibited by law (like race).

5.Jacobson case (Illinois): Contrary case to Weider: Illinois Court ruled there was no need to allow associates who were fired for insisting on compliance with ethical rules to be sue law firms for wrongful discharge. Sometimes its economically advantageous to keep quiet. 6.Scott McKay Wolas: Kelly v. Hunton and Williams: TX: Kelly was a new associate at Hunton and Williams. He seemed to do well and he had good reviews from partners. He noticed that one of the partners, Scott Wolas was misreporting his hours so he tried to tell other partners about it. One of the partners told one of the associates to correct the misreported hours of Wolas. He received bad reviews after he started reporting it. The firm conducts a sham investigation, and finds no fraud. The associates want to report it, but the partners dont think it should be reported. Kelly is encouraged to resign and does so. Wolas ended up disappearing and it turns out he was running a money scheme from his office and stole 30-40 million from clients. Court turned to the Wieder case for ruling. The firm tried to argue that because Kelly was not an admitted attorney yet, Wieder didnt applya. States differ. In Texas, they can fire you. In NY they cant i. See Rule 8.1 Applicants 7.The Photographer Problem 2.2, page 126: Junior partner finds out that senior partner is overbilling big clients. He tells another senior partner, who recommends the junior partner report the overbilling. There is no statute on retaliatory discharge, and the firms partnership agreement doesnt have any explicit language on whistleblowing. Anytime someone says anything, theyre let go. What do you do? i. This was a real case in texas: the junior partner reported, and the middle partner who she reported to, passed it along to the senior misconduct partner. The junior partners work was cut, and she was told to look for work elsewhere. She sued for breach of fiduciary duty, and breach of contract (constructive ousting from the firm). She also files a complaint with the bar association. the bar council starts to investigate but the bar complaint is closed b/c client refuses to participate. In the civil case still going on, junior partner got compensatory and punitive damages for 4 mill for breach of fid duty (reduced later to 237,000). this is later overturned. 1. Texas supreme court: The firm did not owe the partner a duty not to expel her for reporting their overbilling. If you report, you run the risk of being fired. This decision is really scrutinized. Firm eventually went out of business. 2. It seemed like the TX court was giving deference to the harmony of partnership, trying to keep relations smooth between all partners (so this goes against whistleblowing) ii. Rule 8.3: Reporting misconduct: iii. The Kentucky 8.3: A lawyer acting in good faith when he incorrectly reports misconduct is immune from charges, civil or criminal. iv. Rule 1.5 Reasonable Fees: 8.Rule 8.4(c): It is professional misconduct for a lawyer to engage in conduct involving dishonesty, fraud, deceit or misrepresentation 9

2. Civil liability of lawyers:


a.legal malpractice: We use the general term legal malpractice but it encompasses tort claims, breach of K, breach of fiduciary duty claims, etc. They all fall within the umbrella of legal malpractice. i. Tort claim of legal malpractice: The client must assert: 1. lawyer owed a duty 2. lawyer failed to exercise competence and diligence normally exercised by lawyers in a similar circumstance 3. the breach of duty caused harm to plaintiff 4. The client must show that but for the lawyers conduct, the harm would not have occurred (he would have prevailed in the matter in which the lawyer represented him) 5. most common mistakes that lead to malpractice liability: ignores conflicts of interest sue former client for unpaid fee accept any client and any matter that comes along do business with your client practice outside your area of expertise b. malpractice insurance i. Lawyers are not required to carry malpractice insurance but a prudent lawyer should do so. ii. In many states, lawyers are required to disclose whether they have malpractice insurance either to the bar or to clients directly. This information would then be available to the public. iii. What kinds of acts are unlikely to be covered by a malpractice insurance policy? 1. intentional acts including fraud, or other dishonest conduct like billing fraud 2. orders of restitution of legal fees, fines, or penalties 3. orders to pay punitive damages (some policies allow separate purchase for this) 4. conduct of lawyers in other roles like a notary public, title agent, trustee, fiduciary 5. Intra-firm disputes, like liability to former partners or former employees. c.other civil liabilities of lawyers i. advising or assisting unlawful client conduct ii. stealing iii. lying iv. intentional infliction of emotional distress v. violation of regulatory statutes vi. breach of contract vii. Remember, lawyers are bound by law as are ordinary citizens d. disqualification for conflicts of interest i. In deciding motions to disqualify lawyers b/c of alleged conflicts, court do no simply interpret the ethics code (these standards articulate rules for discipline, not disqualification). Many courts follow their own common law standards which may not be the same as the ethics code.

10

3. Criminal liability of lawyers a. Lawyers are not protected from criminal prosecution for white-collar crime 4. Client protection funds a. Client protection funds are state-sponsored programs designed to reimburse clients whose lawyers have stolen their money. Most programs are now called Lawyers Fund for Client Protection. b. They exist in every state c. states do this because in cases of intentional misconduct, malpractice insurance will not cover costs and clients do not get their money. 5. Summing up: What governs lawyers? Moral Certainty: Im doing the right thing. Do not assume certainty in resolving problems. You want to guard against absolutism, and always keep relativism in mind. Morality is what WE believe as right/wrong as people. Ethics is what the system says is wrong:

Chapter 3: The Duty to Protect Client Confidences


1. The basic principle of Confidentiality Rule 1.6: lawyer shall not reveal information relating to the representation of a client unless the client gives informed consent, or the disclosure is impliedly authorized. The lawyer may disclose for these 6 exceptions: prevent reasonably certain death or substantial harm to prevent client from committing a crime or fraud to prevent, mitigate, or rectify injury to ones financial interests to sure legal advice about compliance with the rules to prove claim or defense on behalf of lawyer in a controversy b/w lawyer and client to comply with a law or court order shall not: mandatory, defines proper conduct for purposes of professional discipline. (Comment 14 of Scope) may: permissive and the lawyer has discretion to exercise professional judgment If disclosing under one of the 6 exceptions, the lawyer should first seek to persuade the client to take suitable action to obviate the need for disclosure. the lawyer should not make disclosure adverse to clients interest that is greater than reasonably believed to be necessary to accomplish the purpose. I the disclosure is make in connection with judicial proceeding, it should be made in a manner that limits access to the info. Under the 6 11

exceptions, a lawyer is permitted, but not required to disclose information relating to a clients representation to accomplish the purpose of the exception. a.Protection of information relating to the representation of a client: Information that is protected as confidential: all info relating to the matter on which the lawyer is representing the client, except information that generally known personal information relating to the client that the client would not want disclosed information learned from the client, and information learned from interviews, documents, photographs, observation, or other sources info acquired before the representation begins (like during preliminary consultation) and after the representation terminates Notes or memoranda that the lawyer creates relating to the matter Punishments for failing to protect confidentiality: professional discipline tort liability or contract for breach of duty disqualified from representation of one or more clients, or enjoined by a court from further revelation General discussion of work is fine, with no identifying or personal information. The fact that you are representing a certain client is not confidential, unless the client does not want anyone to know ***Protection of client confidentiality also applies to disclosures by a lawyer that do not in themselves reveal protected information but could reasonable lead to the discovery of such information by a third person. See cmt 4 for Rule 1.6 Social interaction that reveals confidential client info is not allowed The Restatement draws line between disclosures that could harm a client and those that could not. R says whether there is a reasonable prospect of harm to a client depends on whether a lawyer of reasonable caution, considering only the clients objectives, would regard use or disclosure in the circumstances as creating an unreasonable risk of adverse effect either to those objectives or other interests of the client. Dinner with Anna Problem 3.1: Can you tell your friend about the case youre currently working on? You disclosed basic facts of the case, clients first name, name of the bar where incident took place, name of the police officer, opinions about the case, plan of investigation, discovery/litigation. Is this too much to disclose? Yes, she could reasonably go and find out confidential info now (cmt 4) Its not good enough to swear her to secrecy: think about the people sitting around you. It doesnt stop you from speaking broadly about work though. Comment 4 to Rule 1.6: The rule prohibits a lawyer from revealing information relating to the representation of a client. This also applies to disclosures by a lawyer that do not in themselves reveal protected information, but could reasonably lead to the discovery of such information by a third person. The use of a hypothetical to discuss issues relating to the representation is permissible so long as there is no reasonable likelihood that the listener will be able to ascertain the identity of the client or the situation involved. 12

Dinner with Anna Scene 2: Problem 3-2: What if friend asks detailed questions and you answer them? Well, some of what you say may be violations if you answer specifically. In this case, lawyer may have crossed the 1.6 barrier because she gave a lot of detailed answers. Look to see whether one of the exceptions to 1.6 applies. Also, pg 160 says that a lawyer may talk if they are virtually certain that the listeners could not ascertain the identity of the client or if the lawyer could show that there is no reasonable prospect that the discussion would adversely affect the client. This is closer to the norm than the statutory language. You probably went too far- name of bar, name of client, The permissive standard of the Restatement bars revelation of client information if there is a reasonable prospect of harm to the client. b.Protection of information if there is a reasonable prospect of harm to a clients interests i. Unlike the Model Rules, the Restatement (Third) of the Law Governing Lawyers prohibits revelation of confidential information only if there is a reasonable prospect that doing so will adversely affect a material interest of the client or if the client has instructed the lawyer not to use or disclose such information. This allows more room for disclosure than ABA Model Rule 1.6 which prohibits disclosure of any such information except as permitted by the rules. ii. The exercise of discretion and how you communicate is going to vary from lawyer to lawyer, so its hard to establish a black-letter law. c. The bottom line i. Quote from Hazard and Hodes: the line of permissible and impermissible disclosure should be drawn at anonymity. The lawyer should exercise self-restraint and resolve marginal cases in favor of non-disclosure. 2. Exceptions to the duty to protect confidences: a.Revelation of past criminal conduct i. if the crime is over, the lawyer cant benefit from disclosing- the community however can benefit. The lawyer should protect as confidential most information about past criminal activity by clients- if its in the past, you do not disclose unless disclosure is reasonably necessary to prevent reasonably certain death or substantial bodily harm. 1. Reasonably certain: it will be suffered imminently or there is a present and substantial threat that a person will suffer such harm at a later date if the lawyer fails to take action to eliminate the threat. 2. Ex: a lawyer knows that a client has accidentally discharged toxic waste into towns water supply may reveal this info to authorities if there is a present and substantial risk that a person who drinks the water will contract a life-threatening or debilitating disease and the lawyers disclosure is necessary to eliminate the threat or reduce the number of victims. ii. Other exceptions: 1. client gives informed consent 2. the disclosure is impliedly authorized 3. or disclosure is permitted by 1.6(b): 13

a. to the extent the lawyer reasonably believes necessary b. to prevent reasonably certain death of substantial harm c. to prevent the client from committing a crime or fraud that is reasonably certain to result in substantial injury to the financial interests or property of another and in furtherance of which the client has used or is using the lawyers services d. to prevent, mitigate, or rectify substantial injury to the financial interests or property of another that is reasonable certain to result or has resulted from clients commission of crime or fraud in furtherance of which the client has used lawyers services e. to secure legal advice about complying with MRules f. to establish a claim or defense against a client on behalf of lawyer in controversy against client, to establish a defense to criminal charge or civil claim against lawyer based on their conduct in which the lawyer was involved g. to comply with other law or court order. 4. If a lawyer needs to reveal confidential information to protect her own interests, she must take steps to avoid the need for revelation, to limit its scope, or to limit the dissemination of the information. iii.The Missing Persons Problem 3.3: Your defendant reveals during hypnosis that he killed other people in the past. Youre not sure if hes telling the truth- what do you do? a. Robert Garrow (murderer) b. Alert police c. Do a site investigation d. Do nothing about the missing girls Probably best to do nothing- In the real case, the attorney did check out the bodies but did not reveal the information: but you shouldnt do anything- if you dont go, you dont know. 1.6b1, and 1.6b6: look at comment 15, the rule does NOT compel disclosure. What about under the exceptions: complying with law (must report discovering of body): Its permissive, so you still dont have to disclose to prevent future harm/injury: again, its permissive, plus, these bodies are not in danger. theyre already dead. the attorneys did not get in trouble for not reporting, but the court emphasized that an attorney must protect his clients interests, but also must observe basic human standards of decency. Problem 3-4: Missing Persons Scene 2: You actually go to the site, take photos of body. What if one of the deceaseds parents comes to visit you? Do you tell them you know about their daughters death and know where her body is? Rule 1.6 Comment 18: You are held to confidentiality standards even after the lawyer client relationship terminates Real case: atty did not tell anyone and did not want prosecutor to know because it would add on charges- closer to trial, prosecutor tried to sue for obstruction of justice but court ruled for attorney. 14

People v. Belge: real case suing the lawyer after he failed to reveal location of bodies. You should have never let the parents in the office. Problem 3-5: if he escapes, and he once told you where he used to hide out, and he might have been stalking your children. Do you tell? You must use your discretion. Disclosure in this case would be permissible. In an instance like this, you should use your discretion, which may be why the rule is drafted that way. may you tell? yes, if you think the risk to others is great. Rule 1.6a says normally there is a duty but there is an exception under 1.6b Real life, attorney did tell- gun fight ensued and police killed suspect. b.The risk of future injury or death Spaulding (P) v. Zimmerman (D) 1962: no 1.6 rule yet Zimmerman was driving and Spalding was passenger, they wrecked and Spalding sued Zimmerman for injuries. Zimmermans doctor examined Spalding and found an aneurysm, but they didnt tell plaintiffs this (and plaintiffs doctor did not discover it during the initial examination). They settled for not much money, and later the aneurysm was discovered. Canon said it was duty of a lawyer to protect clients confidences The settlement is vacated by the State supreme court. Based on the modern rule 1.6, you could disclose Under the old ABA Canons of Professional Ethics, there was an ambiguity of what confidentiality was- You could try to find a way to disclose- talk to the doctor about telling the plaintiff, or talk to dr. to convince the defendant into disclosing... RULE 3.3: You have a duty to exercise candor toward the tribunal Dinner with Anna Scene 3: Problem 3-6: Youve been doing research on Diet Kolatheres slight statistical evidence to show it causes miscarriages and higher rates of limb deformities in their offspring... Your firm is working in the interest of the food company- can you tell your pregnant friend who drinks Diet Kola? Rule 1.6a: cant reveal info relating to representation of client (youre lobbying for the food company) unless the give consent (which they wont). This doesnt fall into any of the 6 exceptions of 1.6b. 1.6 appears to require that you do NOT disclose, but you would try to find a way to say little...tell her little, tell her you cant say anything else, tell her, but maybe not say anything. Its not reasonably certain death or substantial bodily harm. Look at 1.6a particularly: maybe do NOT disclose Ramifications of choice: You have make a decision about the way you will handle these situations and still have the impact you desire, along with upholding your PR. Comment 6: Disclosure adverse to client: COMPARE this with the brutality case: at some level youre at risk of violating the rule and NOT violating the rule. Maybe this is an excuse in the rules. What if your job is on the line? Conflict b/w practical reality of being fired and ethical duty to disclose...

15

Moral v. ethical (morals: your own system, ethics: systems rules) c. Client frauds and crimes that cause financial harm: lawyers should not facilitate in their clients commission of fraud Rule 1.2(d): a lawyer shall not counsel a client to engage or assist a client in conduct that the lawyer knows is criminal or fraudulent. Comment 10: The lawyer is required to avoid assisting the client (either by engaging in acts or suggesting how the wrongdoing might be concealed). If lawyer discovers client conduct after they take the case, they must withdraw. What conduct constitutes assisting a fraud: Failure to disclose the fact that a lawyer has offered false testimony (court interpretation of Rule 3.3: candor to tribunals). Rule 1.0(d): Fraud: conduct that is fraudulent under the substantive or procedural law or the applicable jurisdiction and has a purpose to deceive. the drafters did not intend to include in this definition merely negligent misrepresentation or negligent failure to apprise another of relevant information. For purposes of this rule it is not necessary that anyone has suffered damages or relied on the misrepresentation or failure to inform. Rule 1.16(a): Sometimes withdrawal alone is insufficient: it may be necessary for lawyer to give notice of the fact of withdrawal AND to disaffirm any opinion, document, affirmation or the like. Rule 1.6b2: Confidentiality can be broken to prevent before conduct Rule 1.6b3: Confidentiality after conduct has occurred. Drafters of the new exceptions allowing revelation of financial crimes and frauds tried to balance three policies: encourage frank comm b/w clients and lawyers to prevent harm to the public protect the integrity of the profession by allowing lawyers to be whistleblowers if their own work is being used to commit crimes or fraud. Rule 4.1(b): bars a lawyer from knowingly failing to disclose a nonconfidential material fact when disclosure is necessary to avoid assisting a clients fraudulent act. Comment 1: misrepresentation can also occur by partially true but misleading statements or omissions that are the equivalent of affirmative false statements. Enron and Sarbanes-Oxley Act: 1.6b2-3 came about because of Enron... Congress passed the act to prevent further episodes of massive corporate fraud. It authorized the SEC to promulgate tough new disclosure rules for professionals, including lawyers, who practice before the Commission and become aware of clients frauds. Lawyers who practice before the SEC or who advise companies regulated by the SEC must report any information about securities fraud to the highest officials of the corporation Relationship b/w Rule 1.6-b-2 and 3, and the Sarbanes-Oxley Act: MR apply more broadly (not restricted to the disclosure of frauds by publicly held companies, nor to lawyers that practice before the SEC)

16

The Sarbanes-Oxley Act could trump state PR, and may REQUIRE disclosure in some cases where revelation would not be permitted under MR 1.6 (b/c federal law preempts state law!) Reeses Leases: Problem 3-7: client is forging documents, copying signatures, but then you talk to them and the client promises to stop doing it. What do you do? 1.6b3: Reveal the fraud? Is revealing the fraud enough? Do we need to do something more? Not revealing and continue to represent: Probably could NOT do that What you should do: You should definitely reveal: presume continuing fraud in a case like this and think that 4.1 mandates disclosure. No matter what, you have to stop representing the client. Not reveal and stop representing them?: 1.6 was not triggered for them b/c the fraud was in the past. Key here is that revelation is discretionary. 4.1 comment 3: Mandatory, so no discretion 1.16 The real case: consulted an expert had no duty to withdraw opinion letters they sent to the bank b/c at that time they were unaware eventually this firm was replaced by another firm, and the new firm was not alerted of any ongoing fraud- later, the old firm was sued and had to pay damages. the consultant who gave advice (a dean) did not get in trouble. How do we balance 1.2, 1.6, and 4.1? Rule 1.6 is permissible, but 4.1 (truthfulness to others) makes revealing a requirement. 4.1 actually serves a limit on the permissive disclosures of 1.6 4.1 is a subset of 1.6, this is a drafting problem. Other relevant rules in revelation of criminal or fraudulent conduct: 8.4 general dishonesty... 3.3 candor toward the tribunal 1.13: duties of a lawyer representing an organization to call attention to crimes and frauds (when someone associated with the business is acting or intends to violate the law) 3.4: prohibits lawyers from hiding or destroying evidence or advising a witness to testify falsely. 1.16-a: requires withdrawal from representation if continued representation would result in violation of the rule 1.16-b: permits withdrawal from representing a client who persists in criminal fraudulent conduct Other ethics rules besides 1.2, 1.6, and 4.1 that allow or require revelation of criminal or Lawyers are NOT protected from civil or criminal liability if they elect not to reveal. The exception to 4.1 is the 1.6 exceptions: 1.6 is a subset of 4.1 Problem 3-8: the investment project Is it ok to buy the last piece of property when you know your client is not going to buy it? 17

maybe they would change their mind so you should be cautious Cmt 5 to 1.8: when you use info to clients disadvantage, it violates your loyalty, The rule does NOT apply to uses which do not disadvantage your client. if they know theyr enot goint to buy it, you can go ahead and purchase it. if they were still considering it, you would need informed consent. Here you dont need it, but you should be safe and get it. Rule 1.8(b): lawyer shall not use information relating to representation of a client to the disadvantage of the client unless the client gives informed consent, except as permitted by rules Comment 5: rule 1.8-b applies when the information is used to benefit either the lawyer or a third person, such as another client or business associate of the lawyer-rule does not prohibit uses that do not disadvantage the client. For example, you learn a gvt agencys interpretation of trade legislation during the representation of one client and you can properly use that info to benefit other clients. Its not a good idea to go into business with a client for several reasons: Problem 3-9: Rat Poison: Your client is being charged with manslaughter for putting rat poison in home where child ate it and died. he keeps slacking on going to the other homes where he used the same poison to remove the pest killers. What do you do? There is both a permissive and mandatory disclosure here. Under 1.6, you could definitely disclose: there is a risk of substantial harm to other children! 4.1: On some level, its a continuing danger: reckless endangerment, and you could be assisting the criminal act if you dont say anything. Solution: You could give him a deadline by when to remove the rat poison units, and if not, you will disclose/report it. In real case, the prosecutor seized clients records and found out the info. Harry did not destroy the information and he handed over the records. A gvt agency went into the homes and removed all the units. Harrys lawyer was able to make a plea bargain..

18

CHAPTER 4: ATTORNEY-CLIENT PRIVILEGE


CONFIDENTIALITY: think about the ethical rules violation, subject to discipline broader than attorney-client privilege PRIVILEGE: think about rule of evidence. violation, results in motion practice/objection Think of this is a narrow subset as confidentiality the privilege is between the lawyer and the client when the client is seeking legal advice or other legal services. So some things are protected by confidentiality, but not privilege. Conversations can be separated into non-privileged and privileged information. Only the conversation is privileged, not the underlying facts Only LEGAL information is protected: CHART ON 219 Ethical Duty to Protect Confidences Source Ethical duty, rule 1.6 Attorney-Client Privilege Common law evidence rule Narrower scope: confidential communication between a lawyer and a client for the purpose of obtaining legal advice Quash subpoena or otherwise exclude the revelation from evidence.

Scope

Information relating to the representation of a client (obtained from any source)

Method of Enforcement

Professional discipline

19

ELEMENTS OF ATTORNEY-CLIENT PRIVILEGE: Communication: privilege covers face-to-face conversations, phone calls, memos, emails, faxes, IMs, etc. Privileged Persons (communications with lawyer agents are also privileged- secretaries, paralegals, investigators). Interpreters are covered too. Communication in confidence: The client must reasonably believe that the communication is confidential Communication for the purpose of seeking legal assistance: Communication is privileged only if the purpose was obtaining legal advice (if they ask for business advice, like investment tips, its not covered) If a lawyer doesnt bill a client for the legal advice, tis still privileged. you dont need an exchange of money If part of a conversation is not about legal advice, that part is not covered under privilege Privilege protects communication from client to lawyer, as well as lawyer to client. When communication is privileged: The lawyer cant be forced to testify cant testify over the clients objective- the privilege belongs to the client and can only be waived by him the client cant be forced to testify the client may waive the privilege, even if the lawyer objects Paper and electronic records of the privileged communication are also protected only the communication, NOT the underlying information is protected CLIENT IDENTITY In general, identity of a client is not privileged. However, law is unsettled on whether identification is protected if disclosure would incriminate the client in the very criminal activity for which the client sought advice in the first place. WAIVER The attorney-client privilege can be waived expressly or by the lawyer is the waiver has been authorized by the client. Waiver by inaction: if client is asked a question regarding privileged info in a deposition and the attorney fails to object, and the client answers, it cannot be undone- privilege was waived privilege can be waived if the client or lawyer reveals the info to a non privileged person. Waiver requires a voluntary act by the client or an authorized agent of the client- so if lawyer on accident reveals privileged info, there has not been waiver. Problem 4-1: Murder for Hire Your client allegedly hired someone to kill his father. when youre talking to him in jail you start to believe he actually killed his father. He agrees to admit guilt at first and then later changes his mind. They want to bring in inmate who overhead admission. Can you assert privilege and exclude the other inmates testimony? yes. This was the only place they could meet so you could argue there was a reasonable expectation of privacy. Arguments that the attorney-client privilege applies: purpose was to deliver legal services 20

client has reasonable expectation of privacy if this is the only place that jail provides to talk which makes sense b/c 6th amendment right to assistance to counsel any communication was about legal strategy Arguments that the attorney-client privilege does NOT apply: there was no expectation of privacy the att-client privilege existed here. The Crime Fraud Exception: There is no privilege if a client seeks assistance with a crime or fraud, even if the No privilege for conversation if the client later uses the advice he received from the lawyer during the conversation to commit a crime or fraud criminal acts/fraud in the past are privileged if a client consults a lawyer about a planned crime but doesnt know that the conduct is criminal, the communication is not privileged. As long as the client has the intention to perform an act that is criminal, even if they dont know its criminal, its not privileged If a client asks about whether a certain act is permitted under the law, its probably privileged- as opposed to a request for advice to help commit a crime or avoid apprehension. Procedure for challenging a claim for privilege: opposing lawyer must request documents based on guesses about what might exist or what it might contain the lawyer could argue for in camera inspection of the correspondence in which the judge reviews the documents privately to decide if they are privileged, urging that the inspection might show intention to commit fraud. example of how important privilege claims can be: the tobacco litigation cases from the 90s. Problem 4-2: The fatal bus crash: you represent estate of man who was injured in bus crash. his insurance wont pay now because they say the cause of death was not the crash. You want to interview the insurance executives about the conversation they had with lawyers: The insurances attorney will likely argue that Insurance- attorney client privilege

applies. This was correspondence between them. The purpose was to deliver legal services. Any letters concerned legal strategy with respect to the claim. There is no evidence that it has been shared with non-privileged persons. And crime-fraud exception does not apply. HOWEVER, YOU SHOULD ARGUE THAT privilege has a narrower scope than confidentiality. If purpose was to make money, they are business records not privileged info. You can also argue that they are deliberately lying because they knew about medical records therefore, fraud exception does apply. Finally, argue for additional exception of wrong-doing so an intentional tort can be within the crime-fraud exception (new legal strategy).
Real case: court looked at the documents, heard both sides, and concluded that the purpose of the crime-fraud exception would be effectuated by applying the exception to torts. So the court thought this was def. something that needed to be looked at, therefore no attorney-client privilege. Diamond v. Stratton 1982

21

The Death of the client Swidler & Berlin v. United States (The Dead Murderer Problem 4-3) Duty of confidentiality survives the death of your client. some people have suggested that we have exceptions (like for criminal prosecutions under federal law what are the concerns? civil liability, harm to family and friends, reputation... Problem 4-3: the dead murderer: Real case is Swidler: Your client is dying from cancer in prison, and he confesses to a murder that would let an innocent man go. Your client doesnt disclose, then dies, and the other def goes on trial in a week and could get the death penalty- should you save the innocent def? 1.6: you could claim it fits under one of the exceptions b/c the other guy is in danger of substantial harm What about under the rules of justice? You might want to find out more information before you disclose (look at what kind of case the prosecution has against the innocent defendant) 8.4 misconduct: professional misconduct to engage in conduct that is prejudicial to the administration of justice. How long does the privilege last? no one really knows. We know that death does not end the privilege unless he gives consent to disclose after his death. Actual case: attorneys sought informal opinion from state ethics committee and the committee told them it was permissible to disclose The scope of privilege for corporations: Upjohn Co. v. United States: KNOW THIS FOR EXAM Attorney-Client Privilege for Corporations (FOR EXAM) Control Group test- limits the privilege to communications from persons in the organization who have authority to mold organizational policy or to take action in accordance with the lawyers advice. The reasoning behind this test is that we want to give people who have responsibility for company the privilege. However, the problem is determining whos in the control group. The Supreme Court said that this test frustrates the privilege because low level employees are not protected and therefore, wont give or reveal any information. Renquist said we want a functional, practical test. He then said that the privilege protects communications not underlying facts or certain groups of people. STILL, this test is used by several states because the subject matter test only articulates federal common law and states make their own evidentiary rule Subject Matter Test- This is the test endorsed by the Supreme Court in the Upjohn case. In Upjohn, a company was bribing countries to do business and the general counsel for the company decided to conduct an internal interview. This information was supposed to remain confidential. However, the company decided to voluntarily disclose some info to the SEC. Then the IRS issued a summons for the internal questionnaires promulgated to members of the company by the general counsel. Upjohn claimed that the questionnaires were privileged but the Sixth circuit applied the control group test. The Supreme Court rejected that test. As stated above, the Supreme Court said that this test frustrates the privilege because low level employees are not protected and therefore, wont give or reveal any information. Corporation conducted internal confidential investigation 22

The sixth circuit limited notion of the privilege for the corporate counsel. The supreme court did not like this and adopted the Upjohn Test: a broader test that. they wanted a rule that reflected the way modern corporations function This case expanded the corporate attorney-client privilege in federal proceedings to communications between lawyers and low level employees who talk with a corporations lawyers to long as the lawyers are gathering information to help guide the companys legal affairs. If they can determine it independently, its considered facts. If not, its communication (so you can ask the client about facts, but not about what legal discussions they had with their attorney). Client cannot be asked what did you say or write to your attorney? The old approach was the Control Group Test and is still used in some states: limits the privilege to communications from persons in the organization who have authority to mold organizational policy or to take action in accordance with the lawyers advice- reason: we want to give people with authority privilege Problem is trying to figure out who fits into this group Upjohn Test: The new subject matter test endorsed by Supreme court for cases based on federal law: extends the privilege to communications with any management or lower-echelon employee or agent so long as the communication relates tot he subject matter of the representation. The McNulty memorandum: page 256**** If a legitimate need exists, prosecutors should seek the least intrusive waiver necessary to conduct a complete and thorough investigation, and should follow a step-by-step approach to requesting information. Prosecutors should first request purely factual information, which may or may not be privileged, relating to the underlying misconduct. Under the McNulty Memorandum, ... Category I: factual information Category II: legal advice Whether or not to give over requests to waive atty-client privilege with corporations: page 257: likelihood and degree to which the privileged information will benefit the governments investigation whether the information sought can be obtained in a timely and complete fashion by using alternative means that do not require waiver the completeness of the voluntary disclosure already provided, and the collateral consequences to a corporation of a waiver McNulty only applies to the Dept of Justice. companies dont have to abide by this- its just guidelines for the government for when gvt should request waiver or atty-lient priv or work product doctrtine. Fill in cases from chapter 4! Problem 4-4: Worldwide Bribery: Read this problem!!! Its category II info, so so you shouldnt just hand it over. Three things you can do:

23

waive privilege and turn over records not waive the privilege (probably best answer: you shouldnt give over conf info everytime you have the slightest threat of prosecution.) middle ground: give some information, not all (this is the worst choice) i.

The pro is that if you turn it over you may be able to negotiate with the prosecutor. So, you may instruct the president to waive the privilege and turn over records. However, you may instruct him that he can give a little and hold some back. That way you are in a better bargaining position ii. The con is that if you turn them over, there is no limit to what the government can investigate or prosecute. Try to get something formal from the government first. Then turn them over. iii. You can also choose a middle ground. Just turn over cover and facts but not release any liability information or legal stuff.
WORK PRODUCT DOCTRINE: Must be in anticipation of litigation. If its a routine practice of the business/client to do something, and then litigation arises, it probably wont be privileged b/c its not in preparation for litigation. Its not absolute. A lot of the information you can obtain from other places. Showing of substantial need or undue hardship can overcome a claim of work product immunity when the allegedly protected material is a witness statement or other ordinary work product. Ordinary work product is that which is compiled by the lawyer, but does not contain the lawyers mental impressions (this is why its not absolute) The doctrine gives stronger protection to work product that reveals the lawyers thoughts, strategies, or mental impressions than it does to other forms of work product. But it doesnt protect underlying facts.

CHAPTER 5: RELATIONSHIPS BETWEEN LAWYERS AND CLIENTS:


Relationships between Lawyers and Clients 24

Choosing Clients: Can a lawyer accept work in an area of law where they have no expertise? Well, rule 1.1 says competence requires the legal knowledge, skill, thoroughness and preparation reasonably necessary for the representation. However, if a lawyer can get up to speed, 1.1 does not prohibit the representation Can a lawyer bill a client for getting up to speed? Yes. So long as it is not excessive. Example A client hired a lawyer to represent her in a custody case. He had 48 years of practicing security law but no experience doing custody. Still, he took the case. He billed her for more than $25,000 and pursued a bad theory of the case. Bar suspended him indefinitely for overbilling and not being competent. How is the relationship formed? No formal agreement is necessary. This means that you must be careful about giving casual advice. A lawyer must also respond if they tell a person they will get back to them If a person is waiting and does not get other counsel or if they miss a deadline because they are waiting, a lawyer can be liable What if a lawyer doesnt want to take a case? A lawyer is under no obligation to take any case but, Rule 6.1 encourages lawyers to do 50 pro bono hours a year Rule 6.2 says that a court may assign a case and you can only reject it for good cause A lawyer cannot discriminate when they are choosing clients Legal Malpractice Elements An attorney client relationship existed That defendant acted negligently or in breach of contract That such acts were the proximate cause of the plaintiffs damages That but for defendants conduct, plaintiff wouldve won case 3.HOW RELATIONSHIP IS ESTABLISHED (SEE TOGSTAD) Togstad v. Vesely, Otto, Miller & Keefe: Togstad had an aneurism and a doctor surgically implanted a clamp in his neck to allow gradual closure of the artery where the aneurism was. Attorney, Miller, gave legal advice, did not qualify it. He said he would get back to her, and never did. he said she did not have a case. Never suggested she should talk to another attorney, and never told her he was not familiar in this area of law. He did not conduct the minimal amount of research.. By the time she went to another attorney, the SOL had run. He gave legal advice and she relied on it. The test for atty-client legal relationship: client sought and received legal advice a reasonable person would have relied on it Rule: Four elements must be shown in this type of legal malpractice case: Atty-client relationship existed; 25

Def acted negligently or in breach of contract such acts were Proximate cause of plaintiffs damages But/for = successful (case within a case see page 129) Apply it: Dont make statements about the law without adequate representation. You should explain the scope of representation to make sure youre covered (1.2) Rule 6.2: Accepting Appointments: you should not accept a client if its likely to result in violation of rules, if its likely to result in unreasonable financial burden on the lawyer, or the client/cause is so repugnant to the lawyer as to be likely to impair the client-lawyer relationship or the lawyers ability to represent the client. Rule 6.1: Limitations on freedom to choose clients: lawyers have duty to provide legal assistance to people who are not able to pay for it. lawyers are encouraged to aspire to provide at least 50 hours/year or pro bono representation. lawyers may be assigned to represent indigent criminal defendants, even if they are not paid for it. a lawyer may not discriminate on the basis of race, religion, nationality, sex, age, disability, or another protected category in decisions about who to represent. 4.LAWYERS RESPONSIBILITIES AS AGENTS: AGENCY LAW (3 AUTHORITIES) a. EXPRESS : when the client says they want you to do something b. IMPLIED : an incident of doing the representation for the client (clients not going to tell you to draft documents and file them, etc.) c. APPARENT: only the acts or statements of client or another principle can justify reliance by a third party. if you dont have express authority, but you see your client doing something that gives you apparent authority. 5.LAWYERS DUTIES OF COMPETENCE, HONESTY, COMMUNICATION AND DILIGENCE A. COMPETENCE, RULE 1.1 1.1: lawyer SHALL provide competent representation to a client. This requires the legal knowledge, skill, thoroughness and preparation reasonably necessary for representation. ***The MacCrate Report Identified ten things and four values that attorneys should have See pge 280-281 Ten Fundamental Lawyering Skills: problem solving legal analysis and reasoning legal research factual investigation oral and written communication counseling negotiation litigation and alternative dispute resolution procedures

26

organization and management of legal work recognizing and resolving ethical dilemmas Four Fundamental Lawyering Values: commitment to values of attaining a level of competence in ones own field of practice and representing clients competently commitment to values of promoting justice, fairness and morality in ones own daily practice participating in activities designed to improve the profession, assisting in training new lawyers, trying to get rid of race biases in profession seeking out opportunities to increase their knowledge and improve skills Problem 5-1: The Washing Machine: Hallmart, a retail appliance store This problem is deceptively simple, which is where some of its complexity comes from. Client doesnt have money and offers a payment plan. We get through this quickly. The temptation and inclination is to accept the stores proposal. Question::: Whether representing this client competently requires further factual and legal research.. Options: Do nothing Payment plan to settle Get rid of this case Do more research...read the contract, under what terms did the client sign the contract? were there any oral communications/agreements made to him? are there warranties? this could lead you to counterclaim. look at the companys history for these claims and also see if they can discharged under state law This does create a problem though b/c client may not be able to pay You need to collect more information before negotiating What should our objective be? If you settled it, you wouldnt be subject to discipline. Youre not being paid anyways, and the client is happy (unless the contract says otherwise) 1.2: Look at the scope of representation: you could limit it by agreement No matter how much the case is worth, you should still do everything you can 1.1: Competence knowledge skill thoroughness preparation COMPETENCE IN CRIMINAL CASES: 6th Amend requires that a criminal def be provided with a lawyer whose work meets at least the minimum standard of being effective Strickland v. Washington: Washington committed several crimes, including 3 murders. The state appointed Tunkey, an experienced criminal atty to represent him in a capital case. Washington confessed to 27

the murder and pled guilty against attys advice. Washington had a sentencing hearing where mitigating evidence could be introduced but Tunkey did not offer character witnesses, etc. Tuneky knew that the judge liked individuals taking responsibility for their actions, so he did this as a strategy. Judge focused on aggravating circumstances. Washington submitted 14 affidavits from people who said they would have testified if asked to do. Theres still a case within a case: Look back at Togstad case. pages 273 and 129. You have to prove that better representation would have made a difference. these are virtually impossible because you have such a high standard of what must be proven RULES: To prove that counsels assistance was so defective as to require reversal of conviction, you must prove two things: counsels performance was deficient. This requires showing that atty made errors so serious that he was not functioning as the counsel guaranteed to def by the 6th amendment. (this is really difficult to show! they have to be totally ineffective) show that the deficient performance prejudiced the defense. This requires showing that the attys errors were so serious as to deprive the def of a fair trial, a trial whose result is reliable. So serious, that we have no faith that it was a fair trial. The judicial scrutiny of attys performance must be highly deferential to the attorney counsel has duty to make reasonable investigations or to make reasonable decisions that make particular investigations unnecessary. an error by counsel does not warrant setting aside judgment if the errors has no effect on judgment. Its not enough that the def show the errors had some conceivable effect on the outcome of the proceeding (b/c almost every act would meet this) APPLICATION: The conduct of the counsel here is not unreasonable. Washington suffered insufficient prejudice to warrant setting aside his death sentence. Tunkeys actions were strategic decisions. DISSENT strongly argues that counsel is afforded too wide of a latitude and that the requirement of acting like a reasonably competent attorney is too vague, its to tell them almost nothing. Its unreasonable for counsel to fail to investigate. Is this reasonable? Was the attorney reasonable? Powell thinks not. Here, unlike in the Washing Machine problem, ones liberty is at stake and perhaps you should do more anyways!s defendant was executed in 1984, 2 months after courts decision. There was a compelling dissent that said lawyer should have brought up everything. after this case, for a long time, only 4 percent of criminal defs won on incompetency charges against their lawyers. B. CANDOR 3.3 AND COMMUNICATION 1.4 Is it ever okay to lie? Questions to ask is the subject trivial or private? is anyone harmed by the lie? is the purpose of lying to protect someone? 28

does the person lied to have a right to know? is there a reason to tell the lie, can the problem be solved without lying if you lie, will it require follow-up lies to cover it up? Problem 5-2: Lying to Clients: lowballing i never reviewed this document who did the work? covering up mistakes blaming others for mistakes Rule 4.1: Truthfulness with persons other than clients Rule 3.1: Honesty toward tribunal CMT 7 to Rule 1.4: lawyers may withhold a psychiatric diagnosis of a client when the psychiatrist indicates that disclosure would harm the client. BUT, the comment does not address whether the atty can lie if directly asked by client about it. Generally, lawyers should be honest with clients and others..(although The rule does not explicitly require lawyers to be honest with their clients) Rule 8.4(c) prohibits attorneys from conduct involving dishonesty, fraud, deceit or misrepresentation YOu have to have knowledge that youre lying in order for it to be forbidden. You can lie in negotiations definitions of fraud vary from state to state. If a lie is not fraud it still may be deceit or misrepresentation. CMT 1 to rule 1.4 also requires reasonable communication b/w lawyer and client as necessary for the client to effectively participate in representation. they cant do this if theyre lying! Lawyers must inform clients of important developments in their case Rule 1.13: an organization as a client: orgs dont lie, people do. RULE 1.4: Consulting/communication lawyer must INFORM CLIENT promptly of any decision or circumstances if informed consent is required for anything reasonably consult with the client about the means by which the clients objectives are to be achieved keep client reasonably informed about status of their matter promptly comply with reasonable requests for information consult with client about relevant limitation on lawyers conduct when the lawyer knows that the client expects assistance not permitted by the Rules Explain matters to the extent reasonably necessary to permit the client to make informed decisions Civil liability for dishonesty to clients: can sue the lawyer in tort for fraud or for breach of lawyers fiduciary duty RULE 2.1: CANDOR: lawyer shall exercise independent professional judgment and render candid advice. In rendering advice, a lawyer may refer not only to law but to other considerations such as moral, economic, social and political factors, that may be relevant to the clients situation.

29

CMT 1: A lawyer should not be deterred from giving candid advice by the prospect that the advice will be unpalatable to the client. PROBLEM 5-3, Torture: Context where all parties considered clients want a forward looking approach, to make torture permissible, or at the very least, makes it a defense that the counsel told them they could do it. One important consideration is that you want to protect your client from future litigation. we must give bad news when necessary. You can give them the narrow, forward looking memo they want, but still be candid and warn against pitfalls. 2.1: Advisor: lawyer shall exercise independent professional judgment and render candid advice. In rendering advice, a lawyer may refer not only to law but to other considerations such as moral, economic, social and political factors, that may be relevant to the clients situation. Comment 1: lawyer should not be deterred from giving candid advice by the prospect that the advice will be unpalatable to the client. Its very rare for a 2.1 to be grounds for violation charge. no one has ever been disciplined under 2.1 5.2b: may be a reasonable resolution of an arguable question The lawyers should edit the memo to reveal doubts about the medicare standard and Youngstown Sheet & Tube Co. v. Sawyer Youre probably violating 3.3 if you dont talk about youngstown Steel case C. DILIGENCE Rule 1.3: A lawyer shall act with reasonable diligence and promptness in representing a client. Common violations: failing to return calls or not filing papers with court on time. CMT 1: lawyer should pursue matter for client despite personal opposition or inconvenience and take whatever measures are required to vindicate clients cause CMT 3: perhaps no professional shortcoming is more widely resented than procrastination Canons used to say you were required to act with zealous advocacy but this was problematic because it did not mean you should act hostile towards opposing counsel

D. WHO CALLS THE SHOTS? Rule 1.2: Scope of representation and allocation of authority b/w lawyer and client Jones v. Barnes: We preserve the lawyers autonomy There is no 6th amendment right for a client to demand which issues to raise on appeal the rule is based on underlying theme of collaborative decisions b/w lawyer and client. Problem 5-4: The Unibomber: You represent a package bomber. On appeal, you think your best argument to save him from the death penalty is to claim he has paranoid schizophrenia. He will not consent to psychiatric exam though and does not want you to argue this. Instead, he wants you only to pursue the procedural arguments (that the search and seizure was illegal). Options:

30

Go with the clients wishes and try to persuade jury that his bombings were necessary to warn and protect society from a greater disaster. terrible idea. Despite clients objection, ask the judge to order a psychiatric evaluation. you may not be able to force him to submit. A possible compromise: trick the client into undergoing examination and then use it. this may be the only real option. In the real case, the attorneys kind of tricked the client and used the evidence. the client was upset and sued the attorneys asking for post-judgment relief, but the court rejected the argument. He argued that the evidence undermined his defense. court said the attorneys were fulfilling their duties. Relevant Rules: 1.2: scope of representation 1.4: communication 1.14: diminished capacity- Maintain a normal client relationship 1.6 might be impliedly authorized to reveal info. 1.1 1.3 8.4 We want to maintain client autonomy and not be overly paternalistic. But who should call the shots? You have to balance Client Autonomy and Paternalism. CLIENTS WITH DIMINISHED CAPACITY: Rule 1.14 Guardianship is only appropriate in extreme cases PROBLEM 5-5: VINYL WINDOWS: You go to clients home because she cant come to the office. She is an elderly widow, the living room is cluttered. She tells you that two salesmen came to her house to work on her windows, and talked her into signing papers, and she went to the bank and withdrew money to give them. When they started working on the windows, she would go to the bank and withdraw payments. When they asked for the final 900, she didnt want to pay it b/c the windows were leaky. Now theyre suing her and she cant recall what she paid, when, etc. We would try to pursuit what she wants, but try to explore everything. Real case: facts were a lot harsher. 1.14 try to have a normal client relationship. here you cant so try to get a guardian ad litembut this is probably not necessary- its too early at this point. would violate client autonomy. talk to neighbors...? balance client autonomy with paternalism do what you can to get her to decide PROBLEM 5-6: You are assigned to represent client- the state is going to ask that the client be admitted for mental care. You discover a procedural error in the commitment petition and go to tell him about it. When you visit him, he is angry, aggressive, and very scary. should you still tell him that you have a way to keep him from being committed? 31

Probably try to get the client released. his actions dont dissolve the relationship and youre not required to withdraw. JUVENILES: children are owed the same duties of undivided loyalty, confidentiality, and competent representation to the child as is due an adult client. PROBLEM 5-7: Child does not really know what she wants. Shes staying with a foster parent now that has a lot of experience, but not with special needs children. Grace wants to return to her mother who abused her (child does not really remember the abuse). Can you respect client autonomy while doing whats in their best interest? you DO NOT have to follow Graces objectives b/c she may not be able to make an informed decision you need to determine if she is capable if making decisions under 1.14 and if so, follow her wishes. some law reviews also say that children need and often expect that an adult will make important decisions for them. being assigned as her guardian would be going too far maybe just act as reporter to court the best option is probably family reunification 1.4 comment 6

TERMINATING LAWYER-CLIENT RELATIONSHIPS: 1.16:Declining or terminating representation: 1.16D: Upon termination, lawyer should take steps to the extent reasonably practicable to protect a clients interests, such as giving notice to client, surrendering papers and property to which the client is entitled, refunding advance payments or payments not used, etc. PROBLEM 5-8: Candid Notes: After you stop representing client, they request their entire file. Their file has a lot of candid notes written on it about impressions of the client, etc. Do you hand it all over? the client does not reasonably need the information in this internal memorandum so you probably would not give it to them. tell the client that you think they want all court documents, etc., but that you are excluding internal documents that you withhold from all clients. These are internal documents intended for internal review so the client does not need them and lawyer can refuse to hand them over. Dont redact because it would raise suspicion. GROUNDS FOR TERMINATION: A client can fire you at any time and for any reason. MUST WITHDRAW: you must withdraw if youre fired. 32

you must withdraw if you are ill or incapacitated, (physical or mental health) or you know your representation would be materially impaired if continued representation would violate ethics rules or the law PERMISSIVE WITHDRAW: no harm to the client Even if material harm will occur, you still may be able to if: lawyer reasonably believes client is acting criminally or fraudulent (this is different from KNOWINg they are, in which case withdraw is required) past use of service for crime or fraud (even if its not currently be used for these purposes) rule 1.16b3 clients views are repugnant or imprudent (their views are very terrible) client fails to meet obligations (like paying; first you must warn the client) unreasonable financial burden client unreasonably difficult to work with (continual failure to appear, missing appointments repeatedly) if you have appeared as the attorney in the case, you cant just withdraw and leave. Even if the lawyer has good cause to withdraw, the court may order the lawyer to continue in which case they must. DUTIES UPON TERMINATION: Fee refund of any fees paid but not yet earned Clients papers and property Client continued to have an obligation to pay fees already earned by the lawyer 1.16d

CHAPTER 6: CONCURRENT CONFLICTS OF INTEREST: GENERAL PRINCIPLES:


CHAPTER 6: CONFLICT Things that impact and can undermine the attorney-client relationship. In the 1940s, 50s, and 60s, it was not uncommon for attorneys to represent opposing clients. Identify problems and try to resolve: withdraw not accept representation is the conflict consentable? is screening allowed? RULE 1.7: Conflict of Interests: Current Clients: SHALL NOT represent a client if it involves a concurrent conflict of interest. 1.7(a) A concurrent conflict of interest exists if: the representation will be directly adverse to another client there is significant risk that representation will be materially limited by lawyers responsibilities to another client, a former client, or a third person or by a personal interest of the lawyer 1.7(b) In spite of a concurrent conflict of interest, a lawyer can represent a client if lawyer reasonably believes he can provide adequate representation to each affected client

33

HOW DO YOU KNOW IF YOU CAN PROVIDE ADEQUATE REPRESENTATION? See checklist of questions on page 364. the representation is not prohibited by law the representation does not involve the assertion of a claim by one client against client represented by the lawyer in the same litigation or other proceeding before a tribunal, and each affected client gives informed consent, in writing. if you dont get the informed consent, you cant represent/continue to represent No matter how big the firm is, and where its offices are, we treat the entity as ONE lawyer. PROBLEM 6-1: INJURED PASSENGERS: TAXI: 373-374: two women in a taxi when it crashes. the women do not know eachother. one just has whiplash, and the other has fractures, more severe injuries. If you were to represent both, you would charge based on contingency fees. we dont have any DIRECT adversity, so then we go to material limitation (like insurance policy caps). Is this a non-consentable case? no. As long as everything is open and clients agree, its okay. What to disclose: explain loyalty, confidentiality and attorney -client privilege advantage/risks of proceeding together how confidential info will be handled/shared among clients come up with an alternate plan incase you can no longer represent both (like one will leave and seek other counsel). You should memorialize their agreement to your representation. Becomes much more complicated when settlement arises. What happens when we get a settlement thats different for the two clients? You can share confidences. You need a contingency plan in case conflict arises.

CONFLICTS B/W CURRENT CLIENTS IN CIVIL LITIGATION: You represent Fred in his divorce. A new client, Mona, comes to you about suing Fred for car accident. This is DIRECTLY ADVERSE. If you were to represent Mona, you would know about Freds assets and values PROBLEM 6-2: I THOUGHT YOU WERE MY LAWYER: One of your firms clients was hit by a bus and shes suing the bus company. Firm also agreed to represent a man in a divorce, who now you know is the husband of Doris (bus accident client)- you didnt realize relationship b/c their last names are different. The firm should at least withdraw from representing the husband Analyzing under RULE 1.7: Try to get informed consent (see what to disclose above) If you cant get informed consent, you will need to get rid of one or both clients (if both have provided confidential information adverse to the other). In the real case, the firm was fired from the personal injury matter when they found out firm was representing spouse. Firm sued for fees for work already performed. Court held firm should have received informed consent, but the firm WAS entitled to some fees. CROSS-EXAMINING A CURRENT CLIENT: 34

You represent a criminal defendant (1), and another person (2) who is charged in unrelated crime. 2 is gong to testify against 1 which means you will have to cross-examine 2. REPRESENTATION OF CO-PLAINTIFFS OR CO-DEFENDANTS IN CIVIL LITIGATION: Several states have held that a driver and passenger have sufficiently divergent interests that they may not be represented by a single atty. The Restatement however takes position that this type of conflict may be waivable by the client after full disclosure. PROBLEM 6-3: INURED PASSENGER 2: You represent 1 and 2, injured passengers (you have already gone through all the procedures in 1.7b to make sure you can represent both). Ones injuries are worse than the other-The insurance company in settlement negotiations hints that he might offer 350,000 to settle more injured case, if youll accept only 50,000 for the less injured. What to do? You need to properly What if you find out one of the clients were drunk, which contributed to the accident? you would need to consult with them b/c theyre adverse now. Its a more serious conflict and if its a cross-claim, its not consentable. How can your representation of one client be materially limited by representing the other? 1.7 QUESTIONS to always ask: What are the clients objectives? How can we achieve them? we know there is a material limitation. Is this situation consentable? This situation would NOT be consentable if both clients wanted the maximum payoff. The economics would say this was consentable. Because of the material limitation, you would need to Consult with your clients and get their consent. COMMENT 6 to 1.7: simultaneous representation for clients who are only economically adverse, does not ordinarily constitute a conflict of interest. PROBLEM 6-4: PRISONERS DILEMMA: Theres two class actions, one for womens prison, and one for facilities for mental retardation. the lawyers for both sides are part of same legal assistance program, different offices (doesnt matter, same entity) There are two different interests: Pelligrino: Womens prison is not adequate, no exercise areas, libraries, etc. OConnr case: Mentally disabled facility is overcrowded. state offers to continue reducing numbers of mental facility, and move women into the mental facility which had better resources, build security there. Is there material limitation? Theres a significant risk that services to one are limited by services to other. you can try to talk to everyone. Look at 1.10 Real case: the court looked at the limited representation, etc., and one issue that came up : did the state make the offer in order to make a conflict to disqualify the legal services firm? Court said no, said the state made good faith effort to negotiate. there was clearly material limitation to the representation. Ultimately, court ruled that there should have been disqualification- eventually, the womens prison was built, but only after years of appeal.

35

TAKING INCONSISTENT LEGAL POSITIONS IN LITIGATION: POSITIONAL CONFLICT: inconsistent arguments on a legal issue in different courts at different times. (Sometimes, even though you are arguing contradicting arguments which could create precedent affecting your other client, positional conflict is okay. you have to determine whether or not its okay.) Factors to consider whether theres a positional conflict that creates a serious problem: whether issue is before trial or app. court whether issue is substantive or procedural the temporal relationship between the matters the practical significance of the issue to the immediate and lon-run interests of the clients involved clients reasonable expectations in retaining the lawyer. PROBLEM 6-5 TOP GUN: Large law firm was representing city in lawsuit against gun manufacturers b/c of high gun crime rate. Other client, a pharmaceutical company, believes that your representation against gun companies could create dangerous precedent for their liability of abused drugs. they threaten to no longer use your firm if you continue in gun case. We cant look at obtaining consent since they are already upset You do NOT have to withdraw Argue there is no real risk/real conflict: This is trial court, not appellate court so there is limited precedent value Globals concerns are hypothetical, not about present tense litigation there is no real proximity between these issues. youve already been working for 2 years, and its 6 months before trial so the city is relying on you and youve already invested much time and money. the gun manufacturer that called global may be playing on his own fears of liability- you may want to thoroughly explain the low risk adverse publicity could result if you withdraw from city, and you might demoralize some of your attorneys. Maybe its a good business decision to withdraw. The noble position is not to withdraw: theres no real material limitation and the city really needs this representation. Look at the factors on page 384 to see if you have a positional conflict that rises to unacceptable level. Actual cause: Weil, Gothsal, Manges: they withdrew from the gun case CONFLICTS INVOLVING PROSPECTIVE CLIENTS: There are obligations even to someone who isnt really a client. RULE 1.18: In consultation, no matter how long it is, you cant reveal info learned during that consultation, except for exceptions under 1.9. 1.18c: See Comment 3!!!

36

The amount of time spent with the prospective client does not matter- its an objective assessment When the attorney receives the disqualifying information, they may still be able to represent client according to exceptions under 1.18d. Comment 5 could also be a way around (exception) to 1.18. It allows lawyer to tell prospective client that the attorney will not be barred from representing another client in the matter so the information disclosed may be used. refer back to TOGSTAD case PROBLEM 6-6: THE SECRET AFFAIR: You have info from prospective client, Maria, that would hurt her. your firm already represents her husband in divorce. Maria is a prospective client under 1.18a what about 1.18d2? the website form she submitted the info on had a small disclaimer Is the firm prohibited under 1.18 from using/revealing info in the consultation? REAL CASE: Court said the website really prevented the formation of a explicit contract with Maria. Second, the disclaimer was sufficient to prevent formation of implied contract. The disclaimer of confidentiality was unclear, and the court said the firm may have to stop representing Nicholas and there was a duty of confidentiality. The information cannot be used. Chapter 6 Recap: Ask yourself: what is the source of the conflict? does it meet the particular rules threshold requirements? is the conflict one that imputes to the entire law organization? if so, can the affected lawyer be effectively screened or isolated? Is the conflict of a type that allows client waiver? If so, what has to occur for the waiver to be effective? Conflicts: Material limitation: as a general rule, conflict of interest exists when a lawyers representation will be materially limited by the lawyers personal interests Waiver: Waiver of general conflict is possible when the lawyer reasonably believes that she will be able to provide competent and diligent representation to the client and the client gives informed consent, confirmed in writing. Business transactions: 1.8a Its required that the clients consent be in writing, that the client be advised in writing of the desirability of seeking and is given a reasonable opportunity to seek the advice of independent counsel, that the transaction be objectively reasonable, that the transaction be in writing and in terms that can be understood by the client and the client gives informed consent in writing

37

Lawyers are prohibited from negotiating for literary rights or media rights based on the subject of representation until the conclusion of representation A lawyer is prohibited from drafting documents that make substantial gifts to the lawyer or the lawyers close relatives, unless the donee is related to the donor.

CHAPTER 7: CONCURRENT CONFLICTS IN PARTICULAR PRACTICE SETTINGS (RULE 1.7)

RULE 1.7: Lawyer shall not represent a client if the representation involves a concurrent conflict if interest. Rule 1.8: Conflict of interest with current client: Lawyer shall not enter into a business transaction with a client or knowingly acquire an ownership, possessory, security, or other pecuniary interest adverse to a client unless: full disclosure of transaction and terms in writing, and its fair and reasonable to the client; client is advised in writing of the seeking and is given a reasonable opportunity to seek the advice of independent legal counsel on the transaction; AND the client gives informed consent in writing (signed by client) to the essential terms or the transaction and the lawyers role in the transaction (including whether the attorney is representing the client in the transaction) lawyer shall now use the info relating to representation of client to the disadvantage of the client unless the client gives informed consent, except as permitted by rules lawyer shall not solicit any substantial gift from client, including testamentary gifts prior to conclusion of representation of a client, lawyers shall not make or negotiate an agreement giving the lawyer literary or media rights to a portrayal/account based on representation lawyer shall not provide financial assistance to a client in connection with pending or contemplated litigation: there are two exceptions. see rule 1.8(e)(1-2) cant accept compensation for representing a client from one other than the client unless client gives informed consent there is no interference with lawyers independence of prof. judgment or the C-L relationship you cannot make aggregate settlement of claims when you rep more than one client unless you have informed consent see rest of rule... 38

REPRESENTING ORGANIZATIONS: RULE 1.13: Organization as a client PROBLEM 7-1: My clients subsidiary: back to facts of representing Doris who was hit by a bus and is suing the bus company. You now find out that the bus company is owned by another client company. Go through factors listed on page 400! first look to see if theyre your client (here they probably are b/c material harm that could result and level of control) next look for direct adversity or material limitation on rep (which we have here) Factors affecting whether a related entity is a client: Joint representation in CRIMINAL CASES is STRONGLY DISCOURAGED. remember that! A conflict can cause a 6th amendment violation. if a trial improperly requires co=representation of criminal defendant, even after you make the objection timely, that is grounds for automatic reversal! PROBLEM 7-2: POLICE BRUTALITY: Alston was the victim of police brutality- assaulted by multiple officers. Tom Babbage is a police officer and pled guilty to charges Two other officers, Chip Stone and Bob Morton, are co-defendants. Unsure of their guiltThe PBA has asked you to represent these two. PBA: provides legal fees and lawyers to represent police officers. This is an institutional entity that is paying, and has broad ranging interests. There is another officer who is also charged, and is a board member of the PBA- he has separate counsel. ISSUES: 3rd party is paying legal fees: can you maintain your independence and provide your best representation or are they going to want to limit fees? In this situation its NON-consentable 1.7-a-2: PROBLEM 7-3: POLICE BRUTALITY SCENE 2: what if everyone consents, they give informed consent theres a common interest then can you take it? PROBLEM 7-4: SCENE 3 what if one of the clients refuses to waive any conflicts? can you fire him to keep the more lucrative client? Most courts really disapprove of selling your client out like this (See hot potato doctrine) What if you find out that one of the officers was in the bathroom at the time and pushed the victim into the bathroom. Consent? You would need to get consent, but you cant revel the info without revealing important info. Real Case: testimony of the other officers was NOT introduced. Attorney relied on theory that Officer Babbage acted along. He pled guilty. Then Stone was convicted and appealed claiming 39

ineffective assistance of counsel b/c of your conflicting interests of PBA and its board member Gutman, and that despite his consent to your representation, the judge should have disqualified you to prevent the violation of his 6th amendment right to effective counsel. (so he waived, but was it consentable??). Court found conflict was non-waivable- the biggest conflict was attys interest in keeping 10 million$ agreement with PBA. the attys self-interest was too strong

RERESENTING FAMILY MEMBERS: Representing both spouses in a divorce: Florida Bar Opinion 95-4 (1997): What if youre representing both husband and wife in preparing will, and after you write it, the H tells you hes changed his will and wants to leave property to mistress. you must withdraw. you are bound to protect Hs confidentiality and at the same time, remain loyal to W. Problem 7-5: The MCarthys: You represent H and W. When clerk files their info away, she misspells their last name. Later, another woman hires firm to represent her in a paternity issue- she is saying the H is the father of her child. Firm doesnt realize they represent H b/c of the typo/misspelled last name. There is a direct conflict of interest here. K&D Represent Hugh (Husband) and Joline (will/estate) K&D (Maureen Carr/Gus Kenney) V. Hugh (Paternity/child support) 1.10 Imputation 1.7: Conflict of Interests: Current Clients Consent is NOT possible here. Can you just drop one client and continue to represent them? You have analyze former clients under Rue 1.9 the lawyers duty of confidence is greater than the duty to communicate to a client info that is relevant to representation The best plan is to withdraw from all three parties. You have a continuing duty to advise Wife in estate planning, REPRESENTING INSURANCE COMPANIES AND INSURED PERSONS The lawyer is being paid by one client (the insurer) to represent both itself and another client (the insured). The insurance defense lawyers dilemma is fundamentally about money b/c the lawyer is being paid by the insurer. If there is a conflict b/w insurer and insured, the lawyer should act in the best interests of the insured, except that the lawyer may not assist client fraid. If they insured is also the lawyers client, the lawyer should try to act in the best interests of both clients. If not possible, they must withdraw from both. if there is a conflict, the insurer will pay the cost of hiring a separate lawyer for the insured. Most jurisdictions have held that if the insurer contests coverage, the insurer has to pay for a lawyer selected by the insured unles there is a prior agreement otherwise. Problem 7-6 Two Masters:

40

Insurance attorney was told by insured not to settle a claim w/o her permission. When he called the adjuster, they said they attorney did not need to call the insured to settle. Ill deal with her- you settle the case... The attorney did not settle and he caught a bunch of hell for it. The insured probably did not want to settle w/o her approval because even though insurance co would pay for it, her premiums would change. The firm was upset because the insurer, the insurance company, was the one paying bills! The third party cant decide the settlement, your client has that right. REPRESENTING PLAINTIFFS IN CLASS ACTIONS There can be internal conflicts in a class action Class actions are OUTSIDE the traditional rules of conflicts. We tolerate a level of conflict in these cases b/c we have hundreds, if not thousands of clients, and theres going to be some internal conflict. The overarching theme is that we try to get broad societal public relief (many of these ppl do not have access to representation). On some level, conflict is a natural part of class actions.

REPRESENTING PARTIES TO AGGREGATE SETTLEMENTS OF INDIVIDUAL CASES Difficulty is getting agreement among people in the aggregate settlement Cmt 13 to 1.8 tries to address this.... You may not practically be able to meet requirements when working wit hundreds of clients...

41

CHAPTER 8: CONFLICTS INVOLVING FORMER CLIENTS


THE NATURE OF CONFLICTS BETWEEN PRESENT AND FORMER CLIENTS See chart on page 437-438 The main concept is page 436 Rule 1.9: a lawyer who has formerly represented a client in a matter shall not thereafter represent another person in the same or a substantially related matter in which that persons interests are marginally adverse to the interests of the former client unless the former client gives informed consent, confirmed in writing ***Comment 3: substantially related matters: if they involved the same transaction or legal dispute or if there otherwise is a substantial risk that confidential factual information as would normally have been obtained in the prior representation would materially advance the clients interest in the subsequent matter. Keep 1.7 in mind .... Try to make a factual determination of whether or not it would hurt the former client. do not undo work youve done for a former client... rules prevent you from switching sides (which is why we ask if its the same matter or substantially same) So you have to look at two things: the past representation, and the current representation Rules dont really provide guidance about how long duty to client lasts: See comment 7 to rule 1.9, then see rule 1.6 Have you really terminated the relationship? Even if you have had a long term client yet have not done work for them in the past year, are they still a client? Yes if you have not formally concluded representation. Problem 8-1: Keeping in touch: You performed work for Almond 5 years ago and now a computer service company wants to hire firm to sue Almond Ask if this is a continuing relationship? Is the newsletter seen as giving advice?? Probably not a present client! firm has tried to contact Almond over the years to see if he needed more services, etc., but he has no contact with firm Almond is definitely a former client (It was Brief, Discreet, and even when the former client was solicited to renew the relationship, the former client always declined) We must apply Rule 1.9 This was a tax opinion issue for Almond before, BUT the factual information you might discover would be relevant to the bill collection by the new client in suing Almond (see 42

Comment 3 to Rule 1.9). 1.9 is PREDICTIVE because you want to protect the client from a conflict... Under the Restatement, the clients reasonable understanding of the scope of representation controls. so you could look to this... factors: brief, one issue long time ago the newsletter wasnt advise In trying to determine if you have a former client, Look at: scope of rep length of years what the client expected See chart on page 444: Fact specific determinations of the lawyers representation: IS the client serious enough that the lawyer cannot go on representing without obtaining consent? See the explanation on page 445. Hot-Potato Lawyer cannot fire a client to get rid of a client and retain the more lucrative client. But there are exceptions: page 455: Material adverse is more narrow: you look at the likelihood of risk and the seriousness of its consequences. EVALUATING SUCCESSIVE CONFLICTS: Are the conflicts serious enough that the lawyer may not go forward w/o clients consent? See chart on Page 444 and explanation through chart on page 445 SAME MATTER: anything that is the subject of representation: litigation, a transaction, a subject on which a client requests advice (or a dispute over document that lawyer drafted) SUBSTANTIAL RELATIONSHIP: Focus is on exploring the facts of a problem and on what a lawyer might have learned during the first matter that could be used adversely to the former client in the second. Problem 8-2: The District Attorney You are D.A. and represented a child a long time ago in an auto accident. Same boy, now 17 yrs old is charged with murder. the pros wants you to withdraw b/c you previously represented him when he was 5 yrs old in the auto accident. Should you withdraw? YES, in real case, the atty had to withdraw. he had information from his prior representation of the kid that could have included relevant medical information. You could have used med info to discredit his case. Is this the same/substantially related? Material adverse? if yes, only way to proceed is with consent of the client. Real case: pros was disqualified Rule 1.9 Maritrans v. Pepper, Hamilton & Scheetz Cmt 6 to 1.7*** Atty Messina, a partner at Pepper firm, represented Maritrans, a business, for a long time. 43

During their relationship of over ten years, Maritrans shared a lot of confidential information about their business practices, as well as info about their competitors Pepper then began representing several of Maritrans competitors Maritrans found out and objected, but Pepper said this was a business conflict not a legal conflict. Pepper and Maritrans negotiated a deal that Pepper would not take on representation of any other competitors, especially Maritrans largest competitor, of Maritrans agreed to let Pepper continue being their attorney and would not represent any more than the four companies pepper was already representing. In addition, the Pepper atty with the most knowledge about Maritrans would not counsel Maritrans, but instead would represent the other four companies, and then 2 different attorneys would represent Maritrans. Chinese wall: attorneys on one side of the wall do not discuss their respective representations with the attorneys on the other side. the firm thought perhaps this was just an economic conflict, not a materially adverse conflict. Then Messina parked the largest competitor with another atty at another firm, and was working on bringing that atty over for partnership to Pepper firm. Comment 6 to 1.7 three main points: injunctive relief: stopping something by preserving the status quo. fiduciary duty: conflict: can you really consent to these situations? What do substantially related and material adverse mean? Rule 1.9 doesnt give us specifics but if you look at the comments, 1, 2, 3, theres guidance on how to look at these terms. Here, there was more than a mere economic conflict. The trial court granted a preliminary injunction. The superior court reversed. This court reverses the superior court. Court says there are times when the danger of revelation of the confidences of a former client is so great that injunctive relief is warranted. This is the case here. There is a civil liability to support an injunction here. If fiduciary duty has any meaning, we cant allow these types of situations. This is conflict that should have been enjoined. DISSENT: consent was given here, and there was a contract under agreed terms of representation, which were not breached! First ask if its substantially related, then ask how it affects the client, and if its material adverse DIFFERENCE BETWEEN 1.9A AND 1.9B: 1.9A: ASK WHETHER THE ATTORNEY COULD HAVE ACQUIRED CONFIDENTIAL INFO IN THE FIRST REPRESENTATION THAT MIGHT BE USED ADVERSELY TOT HE FORMER CLIENTS INTERESTS 1.9B: ASK WHETHER THE LAWYER ACTUALLY ACQUIRED MATERIAL CONFIDENTIAL INFORMATION. (WHERE THE PERSON OR SUBSTANTIALLY RELATED MATTER WAS REPRESENTED BY THEIR FORMER FIRM)

44

To know if a lawyer has acquired material confidences, one must analyze the specific facts relating to the lawyers access to or information about the relevant matter. Comment 6 after Rule 1.9 directs analysts to use certain assumptions Comment 8: if you have not acquired material confidences, you dont need consent. In any situation in which there is a substantial risk that the lawyer would normally have obtained confidences in a former matter that could be used adversely to the former client in a subsequent matter, the lawyer may not be able to handle the second matter unless the former client consents. 1.9C: bars revelation of confidences received from former clients and prohibits the adverse use of such confidences. Problem 8-3: Dysfunctional family: Henry and Joseph co-own HER, Inc. and HER Fashion, Inc. There is a dispute and the ownership is divided. Henry own HER inc- Joseph owns HER fashions Atty resigns, continues to represent HER fashions HER inc is still using the logo. Henrys new atty wants you to withdraw. Joseph wants you to continue representing him for HER fashions. Do you have to withdraw? ANALYSIS: Same? Substantially related? Materially adverse? Consent (Which in this case is not an option) The general counsel would have substantial involvement with the client Real case: atty was disqualified: the obligation of atty to not misuse info obtained in rep is to protect the client.. substantive overlap in work performed for these clients you have too much info, so you must withdraw. he can still rep the corp generally, just not in this action. Problem 8-4: page 480-1 Blurry lines between actually being part of the same firm with atty representing codefendant. This is not the same firm- shared information is a possibility though. There is no framework for preserving confidentiality (the secretary is shared). If the lawyers become partners, you cant make an appearance Collinwood would violate rule 1.9a Wht if they became partners and wanted to rep clients in separate trials? NO- once they become partners, they have fiduciary duties to both clients and cant attack eachother. 4: There would be material adversity there: each would be pointing fingers at eachother, particularly in appeal. Representing current client after dropping one could lead to the use of confidential information to the detriment of the former client.

45

CHAPTER 9: CONFLICTS BETWEEN LAWYERS AND CLIENTS


LEGAL FEES: 1.5: FEES: is a soft rule: allows for much discretion No contingent fees in criminal cases 1.5a: Lawyer shall not make an unreasonable fee. reasonable is determined by a number of factors: These factors are not exhaustive, but allow us to make a determination: Time and labor required likelihood that acceptance of the case will preclude other employment by attorney fee customarily charged in locality for similar services the amount involved and results obtained time limitations imposed by client or the circumstances nature and length of the professional relationship with the client experience, reputation, and ability of lawyer/s performing the services whether fee is fixed or contingent Cmt 1: each factor wont be relevant in every case. Contingent fees SHALL be in writing, signed by client Cmt 7: division of fees: client must agree to the division Brobeck, Phleger & Harrison v. Telex: attorney filed petition for cert, other party withdrew, and the atty charged 1 million dollars the written memo specified that no fee would be paid if petition was NOT filed and there was no settlement.... Court says its not unconscionable: the client was a multimillion dollar company and they wanted to hire a highly reputable firm. Fee was not unconscionable, and fee was not unethically high. The memorandum was very ambiguous: didnt disclose scope of representation didnt break down fees In the Matter of Fordham: Public censure, excessive fees Timothy hired Fordham to represent him for OUI, driving w/ suspended license, speeding, etc. clients should not have to pay for the education of their attorneys Why is this excessive and how do we compare it with Brobeck? Brobeck: sophisticated client Fordham: young client Should the sophistication of the client be a factor in determining a reasonable rate? the results are related to what type of case it is 46

we dont want attorneys learning on the job and then billing their clients for it. Brobeck and Fordham come out completely different! Problem 9-1: apartment suffered 70,000 dollars damage from flood. tenant sues LL to get payment. tenant hires attorney at 250/hour, he ends up working so much that the bill is 60,000 they never really discussed how much time should be spend on case (1.2: scope of representation). there is ambiguity- the client says go and get the result, the atty goes and does the work and spends an inordinate amount of time on it. he worked really hard and did tons of work! can you justify the 60,000 bill? this is a close case. maybe this could have been alleviated with communication: the attorney needed to talk about scope of representation 1.2, communication, 1.4 cmt 5 to 1.5: its proper to define the extent of services in light of clients ability to pay. should not exploit a fee arrangement based primarily on hourly charges by using wasteful procedures Rising Prices: Problem 9-2: Modification of fees Cant just raise fees See 1.5d: any change in basis or rate of fee must be communicated to client. the fee must still meet reasonableness requirement problem gives 4 options for actions to pursue in a rate increase. they would all pass under 1.5d include notice in your next months bills send letter to all clients notifying them of the reasons and when the prospective increase will take effect send letter but also ask clients to assent by returning and signing letter maintain present rate for current client and charge new rate for new clients (most conservative, most advantageous to clients, but not for you. you may not be able to pay for your bills) Maybe you could take a mix of all three, and keep the current rate for clients that object, But this raises fairness issues! so it could be problematic. the first two seem like the best solutions you should look at your contracts NCBEX.ORG: Online Practice Exam

1.5: Division of fees between lawyers not in the same firm: You can divide fees if: its proportional (to the work done) client consents in writing and the fee must be reasonable. Comment 7: referral fees are allowed if you take ethical/financial responsibility for the action A referring lawyer can be disciplined or sued if they dont keep up with it Regulation of hourly billing and billing for expenses: A lawyer billing by the hour may not bill for more hours than she actually worked 47

Lawyers may round up to a MINIMUM billing increment Dont invent hours that were not really worked No profit on costs: a lawyer may not bill for overhead or markup costs No double-billing: a lawyer may not bill two clients for one period of time No billing a second client for recycled work No churning or running the meter: a lawyer may not do unnecessary extra work in order to justify billing more hours No billing clients or the firm for personal expenses or marking up expense receipts Lisa G. Lerman, Scenes from a law firm: article that anonymously describes billing practices at one law firm: Churning: turning out hundreds of Fords instead of a couple Mercedes Padding: Bill more- put more hours down to add meat to the billing rate Invent work Magic number: calculating how many hours a day you need to bill to reach the annual target to get bonuses and vacations, etc. Cranking out forms: Easy billing: read what comes and bill for it Triple billing: if you can spend one hour doing three things, charge 3 clients for the hour Billing off secretaries: Fifteen hours of paralegal time: if the paralegal did more than the 15 hour max allowed for paralegals, the remaining hours would be turned into attorney hours. Hit for .5: if you wake up in your sleep and think of a case, hit .5 hours. Thwarting the auditors: transferring hours on heavy days to hours for lighter days Billing envy: 1.5 (c): Contingent Fees: Are the lawyer and clients interests truly aligned with a continent fee? There is no maximum percentage for contingency fees: they just have to be reasonable Contingent fees must be in writing (to explain details to the client, so they understand how it will operate-will the fee come before/after expenses are subtracted, etc.) in writing signed by client state the method by which the fee is determined including the percentage/s that shall accrue to the lawyer in event of trial, appeal, etc. clearly notify client of any expenses for which the client will be liable whether or not the client is the prevailing party after the matter, atty should provide written statement with the outcome of the matter and showing how the fees were determined Financial assistance to client: RULE 1.8(e): you cnt give financial assistance to client in connection with pending or contemplated litigation except: you can advance court costs and costs of litigation 48

for an indigent client, you can pay their court costs and litigation costs RULE 1.8(i): a lawyer may not acquire a proprietary interest in the subject matter of litigation the lawyer is conducting for a client except permitted liens and contingent fees. if the lawyer purchases a claim from a client and ceases to represent them in collection of that claim, the transaction no longer violates Rule 1.8(i) Problem 9-3: Impoverished Client: Your client is suing employer for firing him-age based discrimination Employer says he was just unable to perform duties you think he could win and might recover 50,000 in damages he has financial problems: pending eviction, no savings, you can pay litigation costs if theyre to be paid back by settlement 1.8e comment 10 YOu cant do this: we dont want clients living off litigation we dont want the atty to have a proprietary interest Real case: a lawyer may pay clients living expenses to avoid the client ahving to settle unfair claims (only in DC) you can help in other ways: help client settle with landlord, set him up in a shelter home, secure a loan against settlement for the claim.... 1.8(d): Publication rights: After the case is over, you can have publishing interests, but not while the case is ongoing. Advance payment of fees and nonrefundble retainer fees: 1.16: you must return any unnused fees page 535: Its controversial to require a nonrefundable advance payment from an individual client with whom the lawyer has no prior relationship classic retainer or general retainer Fee Disputes: Rule 1.8(h): You can ask each client in advance not to sue the lawyer for malpractice, no matter what! YOU MUST be INDEPENDENTLY represented. 1.8(h)(2): Cannot settle claim or potential claim for such liability with an unrepresented client or former client unless they are advised in writing that they should seek independent legal counsel, and give them time to do so. LLP: General form of firms now: Limited Liability Partnership takes away general vicarious liability of partners: youre still liable for your actions, and people you oversee, but not of your partners.

49

Fee arbitration: when you continue to bill but your client is not paying, you have to remember client confidences Lawyer canNOT obtain liens on client property. back to 1.8(i): you dont have lawyers to have proprietary interests in client. HOWEVER, you CAN obtain lien if there is also a conract for it, b/c it then constitutes a business transaction. page 541 Lawyer as custodian of client property and documents Client trust accounts NO CO-MINGLING!!!! You have to keep the clients property separate. Rule 1.15 You must keep complete records of the funds or other property for a period specified in state rules. Prompt delivery of funds or property: When you receive a settlement check or other unds that should be paid to client, you have to notofiy the client and make prompt payment of all funds due to client What if there is a dispute about money or property in lawyers possession? 1.15(e) and (c): if there is a dispute about the amount of the fee, lawyer should distribute undisputed portions and keep the siputed part in the client trust account Creditors: page 549 Unless the creditor has a legitimate claim to the particular funds in the lawyers possession, the lawyers duty is to the client, not the 3rd party business transactions between lawyer and client: RULE 1.8(a): three requirements: see page 553. TEST: questions to ask on page 554: Problem 9-4: if you go overboard with disclosure, he might back otu of the deal are there any special disclosures that are needed? who represents who? Through the transaction, he represents himself, after the transaction, he represents the corporation. You need to disclose that you represent your own interests for the purpose of THIS transaction. If any conflict should ultimately arise, I represent the corporation. In order for Lopez to give consent, you have to disclose your interest, and your status as representative of the corporation 1.8a What to do: tell him he should probably get other representation in writing and give him opportunity to do so explain that you will represent the company not him Real case: Rhode island (atty was former Lt. Gvr) Atty did not make disclosure so the friend just thought the atty was a lender, not really a shareholder. Ct held atty breached his fiduciary duty to corporation- and the Pres of corp could have sued (which he did not do). Ct

50

concluded it was too late for Pres of corp to undo transaction but he allowed Pres to buy out the attorney for his 20% at the date action was filed. No disciplinary action was taken. In disclosure, there HAS to be notion that atty represents the corporation, not the individual. GIFTS FROM CLIENTS: no substantial gifts allowed unless person is related to the attorney (family members, etc) cant prepare instruments for clients giving gifts to the lawyer of his relatives (but you can preprare one for your close relative) Rule 1.8(c) SEXUAL RELATIONSHIPS WITH CLIENT: Not allowed unless the sexual relationship exited before the lawyer-client relationship began. If relationship starts, must get independent counsel (COMMENT 17) DIFFERENCE b/w PERSONAL CONFLICT AND IMPUTED CONFLICT (1.10)*** personal conflicts cannot be imputed to members of the firm. they do not present significant risks of materially limiting the representation of the client by the remaining lawyers in the firm.

Chapter 10: Conflicts Issues for Government Lawyers and Judges: RULE 1.11
Rule 1.11 is the only rule that allows screening Canon of Judicial Conduct: Comment 4 highlights the goal of the rule (balance interests) We want ppl to move back and forth, but we dont want attorneys to use info to help or hurt clients. Relate to 1.9(c) Atty is disqualified if they were involved personally and substantially, unless they have consent. If they dont have consent, the firm can still proceed with representation if the conflicted lawyer is screened and does not earn specific fees from the work. 51

1.11 doesnt really define personally and substantially so the rules look to federal regulations for guidance Personally: directly (includes the participation of a subordinate when actually directed by former Gvt employee in the matter Substantially: the employees involvement must be of significance to the matter, or form a basis for a reasonable appearance of such significance to the matter, or form a basis for a reasonable appearance of such significance. It requires more than official responsibility, knowledge, perfunctory involvement, or involvement on an administrative or peripheral issue Confidential gvt information: Comment 8 Confidential government information: The information that triggers the rule must have been obtained through governmental power (such as threat of subpoena) the gvt must be barred from disclosing it or entitled to assert a privilege against disclosure, AND the info must not be in the public domain 1.11(a) is the gvt lawyer who was personally and substantially involved. We discussed consent Basis for disqualification of former government lawyers:***bottom of page 570 Subsequent work could involve use of confidential gvtal info about a person known to the lawyer in a way that would materially disadvantage that person ( this canNOT be cured by consent) (rule 1.11(c)-type 2) **Bottom of page 570 PROBLEM 10-1: Abraham Sofaer was a professor at Columbia Law, a fed dist. judge, and then the general counsel to the US Dept of State (he developed the legal justification for economic sanctions against libya) and for the US air raid on Tripoli that experts believe the Libyans sought to avenge with the Pan Am massacre. After he left govt, he went to private practice and agreed to represent an atty from Libya about bombing of Pan Am 103. this attorney should have conferred with the state dept Look at comment 10 to 1.11 real case: he got semi-puplic reprimand. he appealed but it was affirmed. Whats the difference between 1.11a (screening, confidential info, consent) and 11.d??? 1.11d: How government lawyers should evaluate conflicts that arise because of work that the lawyer did in a previous private law job (former clients). the lawyer had to get a special license to rep libya (because of an embargo he put in place). if he never served in gvt there would probably not have been a problem. possible problem: he definitely participated personally and substantially in the matter of bombing. in the end he received semi public admonishment which he appealed. it was affirmed that he should be censured. he was not allowed to represent even though he had previously voluntarily withdrwan in response to public response. his involvement was a matter under 1.11. 1.11 comment 10: for purposes of paragraph e of rule, a matter may continue in another form in determining whether 2 particular matters are the same, consider: extent to which the matters involve the same basic facts the same or related parties the time elapsed. the atty failed to ask: the larger question: how will this be perceived? 52

how will the perception affect the client? CODE OF JUDICIAL CONDUCT: 4 CANONS: 1. Impartiality: A judge shall uphold and promote the independence, integrity, and impartiality of the judiciary, and shall avoid impropriety and the appearance of impropriety 2. Competence as a judge: A jude shall perform the duties of judicial office impartially, competently, and diligently 3. A judge shall conduct the judges personal and extrajudicial activities to minimize the risk of conflict with the obligations of judicial office (including full disclosure) 4. A judge or candidate for judicial office shall not engage in political or campaign activity that is inconsistent with the independence, integrity or impartiality of the judiciary

Judges cannot be in discriminative organizations. This is a violation of the third canon. (sexual orientation has just recently been added as a type of prohibited organizational affiliation) Page 584: Scalia scenario: He argued that USSC judges are different- if a judge recuses himself, theres an even number (4 to 4) and issues must be decided. Ultimately the issue was decided 7-2, in favor of cheney. was there a conflict? 10-2: this was an actual case underlying allegation that judge is in the pocket of one of the parties in the suit.

Disqualification of Judges Problem 10-2: A trip to Monte Carlo: You represent Apex, company that lost in law suit against another company. You appealed, and it was overturned- one of the judges had taken a vacation with CEO of Apex during pending of the appeal. Now the other companys lawyer files motion asking the judge to withdraw his vote and recuse himself from participating in any further proceedings. If he does withdraw and recuse, the vote might be tied. Rule 2.11 in the Model Code of Judicial Conduct states that a judge shall disqualify himself or herself in any proceeding in which the judges impartiality might be reasonably questioned. He should disqualify himself even if no party files a recusal motion REAL CASE: two judges recused themselves- even after the re-vote, Apex won. The recusal didnt really help anything. As for the lawyer, you should probably oppose the motion. Problem 10-3: Judges Former Professor: You are an atty for Ahr, a govt official who is a defendant for sexual harassment. You client used to be a law professor. the judge in the case used to be clients student.

53

Should a motion to recuse be filed? You dont wnat to do that unless you really have to. You have to really know wwhat the relationhp b/w judge and client is. If the motion requesting recusal is filed, should the judge disqualify herself? Its a maybeSee Canon 3 See Canon 2B: prohibits judge from allowing family, social, political, or other relationships to influence the judges conduct or judgment Canon 3B(7)(b) allows judge to obtain the advice of a disinterested expert on the law that applies to a particular matter if the judge notifies the parties as to who is consulted and of the substance of the communication and allows the parties to respond. This is the real case of bill clinton. Rule 1.12(b): Page 591: You cant negotiate for employment with any person who is involved as a party or as lawyer for a party in a matter in which the lawyer is participating personally and substantially as a judge or other adjudicative officer. A law clerk to a judge or other officer may negotiate for employment with a party or lawyer involved in a matter in which the clerk is participating personally and substantially, but only after the lawyer has notified the judge/officer. Law clerks are permitted to apply for jobs, even with parties/lawyers involved in matters that are pending before their judges, but only after they notify the judge/officer. Rule 1.12(a) personal and substantial participation standard is the same as that used for conflicts of gvt lawyers in rule 1.11

Chapter 11: Lawyers Duties to Courts:


There are boundaries to loyalty to the client. On one end, you have hired guns, on the other, you have officers of the court mentality. Rule 3.1: A lawyer may not file frivolous lawsuits- but we dont know what frivolous is Comment 2: says facts need not be fully substantiated (this is not a clear black letter rule- its factually determinative). the lawyer may use discovery to develop vital evidence On the other hand, the lawyer must do basic investigation FRC 11(b): similar to 3.1 but more detailed: A partys legal theory must be warranted by existing law or by a nonfrivolous argument for the extension, modification, or reversal or existing law or the establishment of new law. Factual assertions must have evidentiary support or, if specifically so identified, by likely to have evidentiary support after a reasonable opportunity for further investigation or discovery. This standard is subject to widely ranging interpretation Footman v. Cheung (type notes for pages 600Parker v. Vigo County School Corp Jimenez v. Madison Area Technical College 54

Differences b/w Rule 3.1 and FRCP 11: see page 602 Penalties applied to lawyers who bring unsubstantiated suits: Rule 11 penalties Attorneys fees Liability for malicious prosecution Problem 11-1: Visit from Paula Jones: You should investigate and question people at hotel who might have witnessed things track down bodyguard maybe reach out the president- just enough to corroborate what your client is telling you can you file a claim now? once you can find some corroboration, you can file the claim (b/c discovery will allow you to find more later) RULE 3.3: A: Shall not knowingly make a false statement of law or fact or fail to correct a false statement of material fact or law previously made to the tribunal by the lawyer fail to disclose to the tribunal legal authority in the controlling jurisdiction known to the lawyer to be directly adverse to clients position offer evidence that the lawyer knows to be false. if lawyer, his client, or witness called by lawyer has offered material evidence and atty comes to know of its falsity, atty has to take reasonable remedial measures, including, if necessary, disclosure to tribunal See chart on page 606. B: if you know of your client engaging in criminal or fraudulent conduct related tot he proceeding you shall take reasonable remedial measures (including if necessary, disclosure to the tribunal) C: A and B apply ad continue until the conclusion of the proceeding, and apply even if compliance requires disclosure of information otherwise protected by 1.6 Rule 8.4 (c): Misconduct This is what really got President Clinton Misconduct is Broad- It builds on candor toward tribunal, but it applies to ALL conduct. when we talk about lawyers interaction with other people, were pointing to 3.3 candor toward tribunal 4.1 truthfulness in statements to others 8.4: misconduct. Nix v. Whiteside: (There was somthing metallic) Does revelation of client perjury undermine effective assistance of counsel? NO. There is no 6th amendment violation It is elementary that a right does not extend to testifying falsely. How can we ever know whether a client plans to give false testimony? Problem 11-2: Flight from Sudan: You represent man seeking asylum from Sudan. He says he worked for an underground newspaper, Democracy. You are doing work to corroborate his info, and you actually end up 55

getting in touch with the former editor in chief of the Democracy. he has never heard of your client. Should you tell your client what you discovered ? Yes: Communication 1.4a3: keep client informed about important info. You shoudl tell your client about the conversation Can you file the affidavit? Yes, its reasonable to still believe your client Do you ask to withdraw? Not yet. you really want to talk to the client first and figure out whats going on. Did youviolate rule 1.6 by contacting Al-Parah? No, theres an implicit authority to investigate infor for your client. Your client permitted you to investigate and reveal info, so youre not revealing client confidences. You havent made a mistake by doing your job too well. Maybe youre doing your job too well- if you find out all this info, you may have to act on it. but probably not. YOu dont have enough to say this is a lie. REAL CASE: This was a law clinic case. the students were on the verge of filing the affidavit when they found out about the information. they confronted him and he insisted the editor was mistaken, and the students were about to withdraw. the client fires the students and gets new attorney- the students didnt know what to do - Luckily, the new atty requested only the documents that had been filed in court at that time. Client got political asylum. See COMMENT: Lawyers knowledge that something is false can be inferred from the facts.

Problem 11-3: Flight from Sudan, Scene 2: This is NOT material under 3.3 Real case : Governemtn atty found out after end of case and was livid: judge rejected to rehear the case. Judge says it was NOT material and declined to reopen the case. Court refused to re-open case because the witnesss tesitmony was not relied upon. there may be some collateral damage to the legal clinics reputation what about the reputation of the law clinic and the attorney? OJ CASE: Cochran, OJs attorney redecorated OJs house before the jury came over to his house. Was this despicable? I dont think so: its not like youre concealing or replacing material evidence. Youre not changing the facts of the alleged crime. Judges disclaimer:? Problem 11-4: Drug Test Your client had to be tested for Meth, and he tested negative, but the test showed traces of marijuana. You only want the Meth results for the judge, and the lab tech says they can resend you the results with only the meth results- when you get it, you realize she just cut off the bottom portion of the paper. What should you have done when you got the results? Could you just have told your client to go somewhere else and ONLY get tested for meth? RULE 3.4: cant alter or destroy document that has evidentiary value 56

when evidence is submitted, it holds a stamp of authenticity on behalf of the lawyer Most attorneys would say its okay to get a new test. Its ethically okay too its competitive in nature If they request a full test however, thats different and you must comply What if you submit the partial report that the tech sent you. If you move to submit the report for court, youre implying its complete. making it look good would probably require some careful doctoring too. What if the court asks you if this is the entire report? how do you respond? If you just say yes your honor - thats close to perjury Thats what I have, thats what I asked them to screen for: REAL CASE: atty presented the partial documents. he gave the second response (thats what I have , thats what I asked them to screen for). the judge was suspicious of the document. Judge ordered new tests- tests came back negative for everything. The atty was so nervous however, that he revealed everything- judge reported atty to the state that brought criminal contempt charges against the atty. He negotiated it down to a civl charge and paid a 100$ fine. Atty violated 3.3: failed to disclose a material fact to the court. Answer: You should get a new report separately and turn that in 8.4c and d: administration of justice Go through the wrong answers on page 628 Problem 11-5: The body double REAL CASE: Illinois Sup Court: Atty didnt give any notice, Atty put a look-alike person on the stand, and got a MISidentification. then asked to clear the courtroom and told the judge. atty moved for directed virect- the defendant was acquitted HOWEVER< the JUDGE was ticked off! and held atty in contempt: This is a misrepresentation by the inferences of the totality of conduct. went up tto sup court and on 4-3 vote, the atty criminal contempt was upheld in state supreme court for deceiving the judge. This type of behavior undermines the dignity of the court misrepresentation by inference So it was a close cut question! Problem 11-6: Refreshing Recollection How youre supposed to interact with others Other witnesses /plaintiffs tell you that the Superior told them their rates were lower than other companies How far can you go with the witness that doesnt tell you this? Can you ask whether the rep made any statements to her to the effect that their rates were lower than those of other companies? Yes! b/c youre not supplying her with info that she doesnt have Can you tell her that the company settled other cases favorably b/c other borrowers remembered that the rep said their rates are lower? This goes too far because youre kind of telling her the client what to say. violation under 3.4b 57

Can you remind her of what the rep looked like? No. Youre providing her with information she doesnt have. you can show her photos like a lineup Anytime you tell someone/some entity what to say, theres a problem you cant supply them with knowledge they dont have. Can you tell her what others have said? then when she says she remembers, can you tell her to be firm when the testifies? Sure, this is all okay. YOu just cant cross the lines of telling her what to say... refreshing recollection vs. supplying information RULE 3.3 What does counsel or assist mean? Can you coach as long as it doesnt induce false evidence RULE 3.4: Lawyer shall not unlawfully obstruct another partys access to evidence or unlawfully alter, destroy or conceal a document or other material having potential evidentiary value. a lawyer shall not counsel or assist another person to do any such act. This rule does not prohibit all concealment or destruction of evidence but only unlawful concealment. Unlawful means the lawyer already has some obligation to disclose it. add cases from page 635-40 PROBLEM 11-7: CHILD PORNOGRAPHY: you represented one of the most respected churches in town. the choir director worked for the church for 30 years. he loaned his computer to another church employee who saw child porn on the computer. he loanee informed the rector who notified you. The choir director admitted that he owned the stuff and then resigned. There has been no publicity and no charges filed. what do you do with the computer now that you have it? REAL CASE: Connecticut: He took the laptop and destroyed it. indicted under the obstruction of justice under the sarbanes-oxley statute. he eventually pled guilrty for not reporting a felony. this was really a stretch of sarbanes-oxley Unclear whether or not he really needed to report it but it may have been required. Rule 3.4 Comment 2 evidentiary discovery is very important procedural right- dont destroy evidence in some cases you can take temp possession but you might be required to turn over evidence to police, etc. the choir director got 5.5 years. Articles on page 645-50: Austin Sarat: There is usually no consequence at all to an attorney for hiding documents. what matters is winning the case and keeping the client. PROBLEM 11-8: Damaging Documents: Should we turn over these smoking gun documents? Pay attention to the definitional section Can you send them 60,000 documents to overload them but not send them the other 2 bad documents? and draft them a letter saying thatyouve complied?

58

legalistic rationales could cause problems. maybe you should produce everything-- and be overly thorough. If you conceal them, you could say youre not disclosing them and opposing counsel set the parameters of disclosure Real case: They concealed the documents and that was the risk they took. Ifyou conceal a document, its very difficult for it to be uncovered. The opposing counsel caught wind of it and was going to re open. They settled. The firm was sanctioned. It was almost like it wasnt a big deal though- the attorneys who were involved were not fired and at least one of them was promoted to partner. You have to have rational arguments to show why non-disclosure is substantially justified so even though these atys were sanctioned, you almost have to ask yourself wheres the harm? Look at the definition, respond, and then let them ask for more- afterall, thats what requests for documents are. We dont really have a dear doctor letter, and the the other memo is not about Somophyllin RULE 3.3-a-2: Prohibits lawyer from knowingly failing to disclose legal authority in the controlling jurisdiction that the lawyer knows is directly adverse to her clients position, if an opponent has not already informed the judge of the adverse authority. You dont have to disclose persuasive/or secondary authority. Dont have to disclose if its from different jurisdictions EX PARTE PROCEEDINGS: lawyer shall inform tribunal of all material facts known to lawyer to enable the court to make an informed decision, even if adverse to client. This duty overrides the obligation to protect confidences under Rule 1.6. IMPROPER INFLUENCE ON JURY: Lawyers comments to the press: Gentile case: 5-4 decision declaring that disciplinary rule was void b/c it was too vague: attys didnt know what they could or could not say. RULE 3.6: trial publicity: b, 1-6: list of publicity thats ok b-7: specifically in criminal cases c: allows lawyers make a statement that a reasonable lawyer would believe is required to protect a client from substantial undue prej effect of recent publicity. d: if one lawyer in a firm cant make a statement, neither can others in that firm Comment 5: list of likely prejudicial topics generally though safe to give press copy of docuemtns that have been filed in court. GENTILE CASE: those clauses were too vague to determine what the attorney could say. Problem 11-9: LETTER TO EDITOR: can you send this letter? Question is whether it would materially prejudice the proceeding. start with 11-9: look at comment 5.

59

Rule 3.6: Comment 5: Factors 3 and 4 are really important: the evidence of lie detector test and saying your client is innocent could be really prejudicial. In a courtroom these would be okay, but is different in public opinion. take out the :abominable passage and the lie detecotr and innocence parts then submit it Or you could have a citizen submit it- they have greater 1st amen rights REAL CASE: lawyer submitted it as it was: publicly reprimaded (not high consequence b/c he had a great reputation) Subin Article: Professor Subin: on some level, lawyers are truth manipulators In one case, he represented a defendant accused of rape. The def. admitted to Subin that he did rape the girl. In trial, Subin tried to prove that the sex was consensual. Can you cross-examine a witness in a harsh criticizing way, if you know the witness is being truthful? It would be misrepresentation to the court, but it would be a minor misrepresentation, not a major misrepresentation. If you were entirely making stuff up, it would be a different story, but if youre simply putting the prosecution to the test, its proper. As long as youre not manufacturing informaiton, its okay. Subins recommendation: The defense lawyer is limited only to good faith challenges (so if a def. confessed, you would have to find a good-faith basis to pursue a defense).

Statements by Lawyers During Jury Trials: 3.7: Lawyer as witness: A lawyer shall not act as advocate at a trial in which the lawyer is likely to be a necessary witness unless: testimony relates to an uncontested issue relates to the nature and value of legal services rendered in the case, or disqualification of the lawyer would work substantial hardship on the client A lawyer may act as advocate in a trial in which another lawyer in the lawyers firm is likely to be called as a witness unless precluded from doing so by Rule l.7 or Rule 1.9 Comments by lawyers appealing to racial or other prejudice of lawyers: Comment to rule 8.4 says that doing so is a basis for discipline. Rule 3.4: Fairness to Opposing Party and Counsel Page 672 Lawyers duties in nonadjudicative proceedings: Chart 676

Exam Review: Malpractice, what does that mean? Sort answers: based upon problems. Nothing about judicial conduct, but the rest of chapter 12 is free game Judicial conduct: main concern of the 4 canons is impartiality

60

Chapter 12 8.4 general misconduct 4.3 Emergency food stamps: can you have a paralegal deceive someone? its a lie, but on some level its a good lie- its not material. the atty did identify himself after the supervisor got on the line why should you misrepresent in this way? 8.4 4.1 the rules make a distinction between what is contextually true: whether or not something is statement of fact can be deducted from circumstancts 12-2: atty should not misldea DeBello and should make clear who he represents Fox may be asked to sign the written statement but could not say dibello needed him to 12-3: 1.2d: cant assist in activity you know is criminal 4.3(a) shall not lawfully obstruct access to information 4.4: did they receive it anadvertently? 8.4 So what is the right thing to do? destroy copies and never refer to them: if the client is willing to take the chance of being prosecuted, maybe the atty should too. Real case: atty showed the ex husbands atty some of the docuemtns the client had obtained, explained that he knew the ex was probably a molester. On morning of trial to begin, father agreed to give sole custody to mother Had there not been a settlement, the documents may not have been admitted. But the little girls were probably old enough to testify at that point and maybe the documents wouldn have been needed 12-4: p 717: Is there an emergency exception to the rules? can you pretend to be a public defender and negotiate ? this could save lives maybe you can talk to him without obtaining any incriminating info (this is improbable) 3.8: atty shouldnt seek from someone w/o representation that they waive their rights. 4.1 violated 8.4 Violated b/c he make false statements of fact and gave advice to someone whose interests were adverse real case: atty suspended for 3 months for violating 4.1 and 8.4 Suspension was stayed for 12 months (court felt that the atty took advantage of the defenant). the conduct dealt with dishonesty, fraud , deceit. He had to take CLE classes, pay costs of discipline hearing, and had to be supervised when meeting with unrepresented clients 12-5 page 727: arguments jsutifying his statements: maybe hes just identifying a potential problem there sno criminal law that says you cant talk about unpopular clients

61

this notion of prejudice to the justice system is unconstitutionally vague (blatantly prejudical to administartion of justice- what does thi smean?) he should be able to express his own opinion afterall, he is a US citizen Arguments in favor of disciplining him: This is a high ranking official with policy making power and he is articulating a view tha tmight have national an dintl implications he should be held to higher standard b/c status as high-rankingg vt official should he be reprimanded? disbarred? Powell thinks public reprimand was appropriate there was public uproar- ppl thought he was trying to intimidate firms he was forced to resign: nothing reall yhappened and he became a fellow at the heritage foundation Third party interests that affect a lawyers behavior: see article on page 729: Other interests impacting how the law is practied there are community interests. the rules say...you can accept unpopulat cases, doesnt mean you endorse it but there are third party interests that affect these standards- this is the misconception of legal ethics.

CHAPTER 12 LAWYERS DUTIES TO ADVERSARIES AND THIRD PERSONS


COMMUNICATIONS WITH LAWYERS AND THIRD PERSONS DECEPTION OF THIRD PERSON Rule 4.1(a): In the course of representing a client a lawyer shall not knowingly.... make a false statement of material fact or law to a third person.... False statements by clients: If the lawyer knows that her client is lying in her presence to someone other than a court, the attorney must withdraw from representing a client who is using the lawyers services to perpetuate fraud. even if no proceeding has commenced, and the client is not using the lawyers services to perpetrate fraud, the lawyer should interrupt the lying, take her client aside, and advise her to tell the truth. However, if the lawyer does not do so, or the client refuses to take such advice, the rules dont require the lawyer to correct the record. Problem 12-1: Emergency Food Stamps: Attorney had a paralegal call the welfare office pretending to be district manager so the welfare office manager would answer the call. the atty then explained why his client had an emergency need to get food stamps immediately. Rules: 4.1 (is it material)? and Rule 8.4c its certainly misrepresentation, but we think its okay. comment 2 to rule 4.1 62

Apple case, page 685: the limited use of deception is not a violation of 8.4 However, in the Gatti case, the attorney was disciplined for violating a duty to maintain personal integrity is it okay for us to have this exception? RESTRICTIONS ON CONTACT WITH REPRESENTED PERSONS Rule 4.2: See comments 1 and 7 In representing a client, a lawyer shall not communicate about the subject of the representation with a person the lawyer knows to be represented by another lawyer in the matter, unless the lawyer has the consent of the other lawyer or is authorized to do so by law or a court order you do not want to undermine their atty-client privilege. Harvard issue page 699: they were mere witnesses, and some of them had supervisory authority. comparison of corporate atty-client privilege and rule 4.2: chart on 698 focus on comment 7 to rule 4.7: interviewing employees of the corporation. There are people you cannot interview an employee who supervises, directs or reularly consults with the orgs lawyer concerning the matter has authority to obligate the organization with respect to the matter is one whose act or ommission in connection with the matter may be imputed to the organization fo rpurposes of civil or criminal liability Messing, Rudavsky, and Weliky v. President and Fellows of Harvard College: Rule 4.2 the rule used to have an admission rationale which was dropped when the rules were advised. why? it was overly broad- anyone would fit into the category (anything could be an admission if it falls within scope of your employment) court rejects admission standard and adopts an imputation view Here, the employees did not fall within the category and could therefore be contacted. see page 701 for the main holding RESTRICTIONS ON CONTACT WITH UNREPRESENTED PERSONS Rule 4.3: lawyer shall not imply or state that the lawyer is disinterested when the lawyer knows or reasonably should know that the unrepresented person misunderstands the lawyers role in the matter, the lawyer shall make reasonable efforts to correct the misunderstanding. The law shall not give an unrep. person legal advice, other than to secure counsel, if the lawyer knows or reasonably should know that the interests of such a person are or have a reasonable possibility of being in conflict with the interests of the client. chart on 705

63

Comment 1: Sometimes a lawyer will have to identify their client to the unrepresented person, and explain that the client has interests opposed to those of the unrepresented person. Comment 2: lawyer can negotiate terms of a transaction or settle dispute with an unrepresented person. So long as the lawyer has explained that the lawyer represents an adverse party and is not representing the person, the lawyer may inform the person of the terms on which the lawyers client will enter into an agreement or settle a matter; prepare documents that require that persons signature and explain that the lawyers own view of the meaning of the document or the lawyers view of the underlying legal obligations. page 705: the major thing is that when youre talking to an unrepresented person, you cant leave the impression that their interests are somehow being safeguarded. the only advice you can give them is to go get independent counsel. Problem 12-2: The Complaining Witness: Can you go visit her? Yes, under 4.3 what do you have to tell her? you have to make sure she knows you do not represent her, her interests, etc make sure you dont create the misunderstanding that you represent anyone other than the defendant what about signing a medical release form? You cannot tell her she NEEds to sign it. Rule 4.4: respect for Rights of third Persons: Rule 4.4: shall not use means that have no substantial purpose other than to embarrass, delay, or burden a 3d person, or use methods of obtaining evidence that violate the legal rights of such a person lawyer who receives a document relating to a client on accident should notify the sender. all the rules require is that you notify the sender what else you do with it is a matter of law in your particular jurisdiction. its beyond the scope of the rules... We dont want unwarranted intrusion into things that should be private Comments 1, 2, 3. Problem 12-3: The Break-in your client broke into her ex-husbands house and took documents as proof that he molested their girls. she photocopied all the documents and brought them to you. what do you do with them? you didnt technically assist but its not inadvertent b/c you know about it 8.4 you could try to ask the girls about them to get more real case: atty did not destroy, just showed the ex-husbands atty some of the documents. atty said she knew the ex was a child molester. the exs atty was shocked. in court, the ex did not contest full custody. even if they had not reached this settlement, you could go the route of asking the girls to get more information leading to a warrant and uncovering of the evidence.

64

the atty never even asked where the documents came from bc he was so shocked at them DUTY OF PROSECUTORS Prosecutors are attys so are held under the general rules, as well as addtl special rules b/c they represent the state Footnote on page 713: FN 53 Page 714: Prosecutorial midconduct UNDERCOVER INVESTIGATIONS McDade Amendment: page 716 an atty for the gvt shall be subject to state laws and rules, and local federal court rules, governing attys in each state where such atty engages in that attys duties, to the same exten and in the same manner as other attys in that state. The enactment of this amendment however, seems to have had little effect on the ability of federal prosecutors to direct indictment communications with a represented suspect Problem 12-4: Prosecutors masquerade: is there an emergency exception to the rules? Weigh the emergency exception against: 5th and 6th amend? you could try to avoid getting self-incriminating statements. Rule 3.8 (its really the analog of 3.1) Comment 2: the prosecution should not seek from a defendant that has no representation a waiver of legal rights 4.1: false statements 8.4: false statements 4.3: are you misleading this client about your role? (misleading an unrepresented suspect) Real case: atty went ahead with the plan. the public took the viewpoint that the atty was a hero for doing this- editorials were published, etc. (Denver). There was tremendous public support for impersonating. However, lawyer violated rule 4.3 and 8.4 and was suspended for 3 months. he was required to retake the MPRE, take 20 hours of CLE on ethics. anytime he spoke to an unrepresented person, he had to be accompanied by his supervisor. he appealed but it was affirmed. He wasnt fired anything, continued to be senior DA. REQUIRED INVESTIGATION BY PROSECUTORS BEFORE CHARGES ARE FILED CONCEALMENT OF ECULPATORY EVIDENCE Nifong was disbarred in this case. UNRELIABLE EVIDENCE Unreliable evidence: just b/c something is scientific evidence, our reliance on this may be unreliable. ENFORCEMENT CONDUCT PREJUDICIAL TO THE ADMINISTRATION OF JUSTICE Problem 12-5:

65

Charles Stinson was on a radio show and he made comments about the attorneys who represented detainees at Guantanamo Bay. you are on an assistant bar counsel for the states bar. what should you do? STOPPE DHERE, finish chapter 12 and talk bout judicial conduct ARE LAWYERS REALLY TOO ZEALOUS? start page 699, finish chapter

Judicial canons: Impartiality is the central theme of all 4 canons. They define the judges role Canon 1: uphold and promote judicial independence/impartiality Canon 2: shall perform duties impartially, competently, and diligently Canon 3: avoiding risk of conflict with the obligations of the office Canon 4: inconsistent with judicial independence, etc Exam: dont need to mention numbers for rules and comments when can personal conflicts be imputed under rule 1.10? Look at comment 3: youre not going to impute a personal bias to an entire firm (Ex: personal conflict of inability to represent KKK- everyone else at the ACLU is not forbidden). The lawyer just wont work on the case. what if you own the corp that your firm is going to sue? now that is imputed to the entire firm. 8.3: Substantial question should be evaluated by the seriousness of the offense. there is some discretion, and it is permissive- not mandatory. Model Rules apply to UNadmitted lawyers 1.6 and 1.2: when would something be impliedly authorized to reveal confidential? the atty is the clients agent- if you needed to know more aboutyour client, you coudl reveal confidences to findout more about your case- do so using a hypothetical. (cmt 4 to 1.6) Foreseeability: Restatement comes into play. crime/fraud: OVerall, the rules try to deal with public harm. 1.6 is really concerned with disclosing confidential info. A-C privilege: Look at the content of the information and see how it relates to the AC privilegecertain facts are going to be well known anyways. Diminished capacity: is it permissive or mandatory? its permissive. Shall TRY to maintain a normal relationship... The problem with the old lady try to make it a normal relationship

66

dont impose too much but eventually you may have to make some determinations (may take reasonably necessary actions) the rule is designed to TRY to maintain th eatty-client relationship. Could this lead to a mandatory obligation? Possibly yes, at some point. Problem 7-1 my clients subsidiary: lawyer cannot represent the company unless they reasonably believe the client would not be effected, but under the facts of this problem, thats probably not possible Informed consent: you should try to get it in writing. this is different if youre entering transaction with client- then you MUSt have it in writing. Problems: 5-5: p 332: Rule 1.14 if reasonably possible, maintain a normal relationship with her. there may come a time when you cant, and we would try to direct her (not make her) 5-6 can you sue the client? Yes, but thats not the point of the question. page 334- you should look at getting client release, try to have normal relationship, possibility of terminating relationship 1.16. look at comment 6 to 1.14- all of the things you should take into consideration 7-2, 7-3, 7-4: that problem is baed on the Abner Louiana case in NY: Key text is Independence then consier 7-2: is there a risk that rep would be materially limited? Yes. atty being in position of wanting the contract renewed... 7-3; can you drop one client ? Hot potato doctrinel courts generally disapprove of that 7-4: reaches point where it may be non-permittable. can they previal under stickland standard? Did the atty sell his client out? is he trying to keep the 10 mil contract? 9-3 page 553: Looked at whether or not there was authority for a non-literal reading of the rule- 1.8 prohibits paying living expenses. Comment 10. you really canNOT provide assistance. the facts were heart wrenching but too bad. maybe try to get emergency assistance from a program 10-2 page 586: USSC ultimately said the judge should have recused himself. The attorney should not join in the motion but you should NOT oppose it either- your epursuing justice as an officer of the court. 10-3: p 587: Clinton was sancioned for lying under oath in his deposition Clintons atty had to disavow some of the statements the president made- that started the ball rolling if lawyer has prohibited conflict b/w two current clients: cmts 2 and 4 to 1.7: generalyl absent consent you cant Screening is required gvt to private and private to gvt. No such screening for private firms (look to local rules). remember the spectrum of confidentality regarding gvt employment. some jurisdictions provide for screening. Successive conflicts: when a new lawyer comes into firm 1.9b1 and b2: You have to overcome a presumption that you have acquired info from former rep that is adverse to your current client.

67

making a prediction see comment 3 to rule 1.9: you presume that the atty has information that can be used against the client. this is a high standard so you have to overcome the presumption you have to maintain confidences even though its former representation you cannot UNDO work youve done for the client same or substantially related matter materially adverse

68

You might also like